Download as pdf or txt
Download as pdf or txt
You are on page 1of 65

MEDSURG- 1

SAS #1 4. On the morning of Mrs. Sy’s planned


cholecystectomy, she awakens with a pain in
her right scapular area and thinks she slept in
1. As a nurse, what is the importance of a poor position. While doing the pre-op check list
thorough preoperative assessment? you note that on her routine CB report her WBC
is 15,000. Your responsibility at this point is:
a. To identify and correct problems before
surgery and establish a baseline for a. To notify the surgeon at once; this is an
postoperative comparison elevated WBC indicating an inflammatory
reaction
b. To save time doing an assessment after the
patient returns from surgery b. To record this finding in a prominent place on
the pre-op checklist and in your pre-op notes
c. To provide the doctor with information that
may have been missed during the preadmission c. To call the laboratory for a STAT repeat WBC
assessment d. None. This is not an unusual finding
d. To ensure that postoperative complications 5. Mrs. Sy is scheduled for surgery 2 days later
don’t occur and is to be given atropine 0.3 mg IM and
2. Before administering preoperative Demerol 50 mg IM one hour preoperatively.
medication to a client, nurse Jonalyn should Which nursing actions follow the giving of the
plan to: pre-op medication?

a. Verify the consent a. Have her void soon after receiving the
medication
b. Check the vital signs
b. Allow her family to be with her before the
c. Have the client void medication takes effect
d. Remove the client’s dentures c. Bring her valuables to the nursing station
3. A client with Cataract is about to undergo d. Reinforce pre-op teaching
surgery. Nurse Princess is preparing plan of
care. Which of the following nursing diagnosis is 6. A patient is admitted to the same day surgery
most appropriate to address the long term need unit for liver biopsy. Which of the following
of this type of patient? laboratory tests assesses coagulation? SATA.

a. Anxiety related to the operation and its a. Partial thromboplastin time.


outcome b. Prothrombin time.
b. Sensory perceptual alteration related to lens c. Platelet count.
extraction and replacement
d. Hemoglobin
c. Knowledge deficit related to the pre-
operative and post-operative self-care

d. Body Image disturbance related to the eye 7. A client with a perforated gastric ulcer is
packing after surgery scheduled for emergency surgery. The client
cannot sign the operative consent form because 10. A nurse is reviewing the physician’s order
he has been sedated with opioid analgesics. The sheet for the preoperative client, which states
nurse should take which of the following that the client must be on nothing per mouth
actions in the care of this client? (NPO) status after midnight. The nurse would
clarify whether which of the following
a. Obtain a telephone consent from the family
medications should be given to the client and
member witnessed by two persons.
not withheld?
b. Obtain a court order for the surgery.
a. Conjugated estrogen (Premarin)
c. Send the client to surgery without the
b. Atenolol (Tenormin)
consent form being signed.
c. Cyclobenzaprine (Flexeril)
d. Have the hospital chaplain sign the informed
consent immediately d. Ferrous sulfate

8. A preoperative client expresses anxiety to the


nurse about the upcoming surgery. Which of
SAS #2
the following responses by
1. Nurse Jay is preparing to change a client’s
the nurse is most likely to stimulate further
dressing. The statement that best explains the
discussion between the client and the nurse?
basis of surgical asepsis that nurse will perform
a. “I will be happy to explain the entire surgical in this procedure is:
procedure to you.”
a. Keep the area free from microorganisms
b. “Let me tell you about the care you’ll receive
b. Protect self from microorganisms in the
after surgery and the amount of pain you can
wound
anticipate.”
c. Confine the microorganisms to the surgical
c. “If it’s any help, everyone is nervous before
site
surgery.”
d. Keep the number of opportunistic
d. “Can you share with me what you’ve been
microorganisms to a minimum
told about your surgery?”
2. Function of a circulating nurse is:
9. A nurse is preparing the client for transfer to
the operating room (OR). The nurse should take a. Assists the scrub nurse and surgeon
which of the following
b. Draping the client with sterile drapes
actions in the care of this client at this time?
c. Wear sterile gloves, gowns, caps
a. Administer all the daily medications.
d. Handling sterile instruments and supplies
b. Ensure that the client has voided.
3. Which among the following are not
c. Verify that the client has not eaten for the members of the sterile team in the operating
last 24 hours. room? Select that all apply:
d. Practice postoperative breathing exercises. 1. Surgeon

2. Anesthesiologist
3. Radiation technician d. To prevent cardiac arrhythmias

4. Scrub nurse 7. What is the most important postoperative


instruction nurse Nikki must give to Mrs.
5. Circulating nurse
Serrano who has just returned from the
a. 1, 2, 3 operating room after receiving a subarachnoid
block?
b. 2, 3
a. "Avoid drinking liquids until the gag reflex
c. 2, 3, 5 returns."
d. 3, 4, 5 b. "Avoid eating milk products for 24 hours."
4. What is the responsibility of the nurse c. "Notify a nurse if you experience blood in
regarding informed consent? your urine."
a. To explain the surgical options d. "Remain supine for the time specified by the
b. To describe the operative procedure to be physician."
done 8. The common anesthesia used for
b. To explain the operative risks appendectomy is?

d. To witness a patient’s signature a. Spinal

5. Which of the following is the primary b. General


responsibility of the nurse before surgical c. Caudal
operation?
d. Hypnosis
a. Taking the vital signs
9. The client returns from surgery with a blood
b. Explaining the procedure pressure of 90/50, pulse 132, and respirations
c. Obtaining the permit 30. Which action by Nurse Jo should receive
priority?
d. Checking the lab works
a. Continue to monitor the vital signs
6. Mr. Alejandro is scheduled for a
prostatectomy and the anesthesiologist plans b. Contact the physician
to use a spinal (subarachnoid) block during c. Ask the client how he feels
surgery. In the operating room, the nurse
positions the client according to the d. Ask the nursing assistant to continue the
anesthesiologist's instructions. Why post-op care

does the client require special positioning for 10. An additive given to potentiate and
this type of anesthesia? prolong the effects of regional anesthesia is:

a. To prevent confusion a. Lidocaine

b. To prevent seizures b. Epinephrine

c. To prevent cerebrospinal fluid (CSF) leakage c. Digoxin


d. Lanoxin d. When the patient can be aroused and
recognizes where he or she is

5. What is included in the routine assessment


SAS #3
of the patient’s cardiovascular function on
1. What does progression of patients through admission to the PACU?
various phases of care in a post anesthesia
a. Monitoring arterial blood gases
care unit (PACU) primarily depend on?
b. Electrocardiographic (ECG) monitoring
a. Condition of patient
c. Determining fluid and electrolyte status
b. Type of anesthesia used
d. Direct arterial blood pressure monitoring
c. Preference of surgeon
6. With what are the postoperative respiratory
d. Type of surgical procedure
complications of atelectasis and aspiration of
2. Priority Decision: Upon admission of a gastric contents associated?
patient to the PACU, the nurse’s priority
a. Hypoxemia
assessment is
b. Hypercapnia
a. vital signs.
c. Hypoventilation
b. surgical site.
d. Airway obstruction
c. respiratory adequacy.
7. To prevent airway obstruction in the
d. level of consciousness
postoperative patient who is unconscious or
3. How is the initial information given to the semiconscious, what will the nurse do?
PACU nurses about the surgical patient?
a. Encourage deep breathing
a. A copy of the written operative report
b. Elevate the head of the bed
b. A verbal report from the circulating nurse
c. Administer oxygen per mask
c. A verbal report from the anesthesia care
d. Position the patient in a side-lying position
provider (ACP)

d. An explanation of the surgical procedure


from the surgeon

4. To prevent agitation during the patient’s 8. Priority Decision: To promote effective


recovery from anesthesia, when should the coughing, deep breathing, and ambulation in
nurse begin orientation explanations? the postoperative patient, what is most
important for the nurse to do?
a. When the patient is awake
a. Teach the patient-controlled breathing
b. When the patient first arrives in the PACU
b. Explain the rationale for these activities
c. When the patient becomes agitated or
frightened c. Provide adequate and regular pain
medication
d. Use an incentive spirometer to motivate the a. Take preventative antibiotics, as ordered.
patient
b. Gargle with warm salt water regularly.
9. While assessing a patient in the PACU, the
c. Dress herself and her infant warmly.
nurse finds that the patient’s blood pressure is
below the preoperative baseline. The nurse d. Wash her hands frequently.
determines that the patient has residual
vasodilating effects of anesthesia when what is 2. The nurse is explaining the safe and
assessed? effective administration of nasal spray to a
patient with seasonal allergies. What
a. A urinary output >30 mL/hr. information is most important to include in
this teaching?
b. An oxygen saturation of 88%

c. A normal pulse with warm, dry, pink skin


a. Finish the bottle of nasal spray to clear the
d. A narrowing pulse pressure with normal
infection effectively.
pulse
b. Nasal spray can only be shared between
10. Priority Decision: A patient in the PACU has
immediate family members.
emergence delirium manifested by agitation
and thrashing. What should the nurse assess c. Nasal spray should be administered in a
for first in the patient? prone position.
a. Hypoxemia d. Overuse of nasal spray may cause rebound
congestion
b. Neurologic injury
3. A patient visiting the clinic is diagnosed with
c. Distended bladder
acute sinusitis. To promote sinus drainage, the
d. Cardiac dysrhythmias nurse should instruct the patient to perform
which of the following?

a. Apply a cold pack to the affected area.

b. Apply a mustard poultice to the forehead.

c. Perform postural drainage.

d. Increase fluid intake.

4. The nurse is creating a plan of car for a


SAS #4
patient diagnosed with acute laryngitis. What
1. The nurse is providing patient teaching to a intervention should be included in the
young mother who has brought her 3-month- patient’s plan of care?
old infant to the clinic for a well-baby checkup.
a. Place warm clothes on the patient’s throat,
What action should the nurse recommend to
as needed.
the woman to prevent the transmission of
organisms to b. Have the patient inhale warm steam three
times daily.
her infant during the cold season?
c. Encourage the patient to limit speech 8. The nurse is caring for a patient whose
whenever possible. recent unexplained weight loss and history of
smoking have prompted diagnostic testing for
d. Limit the patient’s fluid intake to 1.5 L/day.
cancer. What symptom is most closely
5. A patient comes to the ED and is admitted associated with the early stages of laryngeal
with epistaxis. Pressure has been applied to cancer?
the patient’s midline septum for 10 minutes,
a. Hoarseness
but the bleeding continues. The nurse should
anticipate using what treatment to control the b. Dyspnea
bleeding?
c. Dysphagia
a. Irrigation with a hypertonic solution
d. Frequent nosebleeds
b. Nasopharyngeal suction
9. The campus nurse at a university is assessing
c. Normal saline application a 21-year-old student who presents with a
severe nosebleed. The site of bleeding appears
d Silver nitrate application
to be the anterior portion of the nasal septum.
6. The nurse is planning the care of a patient The nurse instructs the student to tilt her head
who is scheduled for a laryngectomy. The forward and the nurse applies pressure to the
nurse should assign the highest priority to nose, but the student’s nose continues to
which postoperative nursing diagnosis? bleed. Which intervention should the

a. Anxiety related to diagnosis of cancer nurse next implement?

b. Altered nutrition related to swallowing a. Apply ice to the bridge of her nose
difficulties
b. Lay the patient down on a cot
c. Ineffective airway clearance related to
c. Arrange for transfer to the local ED
airway alterations
d. Insert a tampon in the affected nares
d. Impaired verbal communication related to
removal of the larynx

7. The home care nurse is assessing the home


environment of a patient who will be
10. The nurse is conducting a presurgical
discharged from the hospital shortly
interview for a patient with laryngeal cancer.
after his laryngectomy. The nurse should The patient states that he drinks
inform the patient that he may need to approximately six to eight shots of vodka per
arrange for the installation of which system in day. It is imperative that the nurse inform the
his home? surgical team so the patient can be assessed
for what?
a. A humidification system
a. Increased risk for infection
b. An air conditioning system
b. Delirium tremens
c. A water purification system
c. Depression
d. A radiant heating system
d. Nonadherence to postoperative care D. Albuterol nebulizer

4. When teaching the patient about going from


a metered-dose inhaler (MDI) to a dry powder
SAS #5
inhaler (DPI), which statement by the patient
shows the nurse that the patient needs more
teaching?
1. While assisting a patient with intermittent
asthma to identify specific triggers of asthma, A. “I do not need to use the spacer like I used
what should the nurse explain? to.”

A. Food and drug allergies do not manifest in B. “I will hold my breath for 10 seconds or
respiratory symptoms. longer if I can.”

B. Exercise-induced asthma is seen only in C. “I will not shake this inhaler like I did with
individuals with sensitivity to cold air. my old inhaler.”

C. Asthma attacks are psychogenic in origin D. “I will store it in the bathroom so I will be
and can be controlled with relaxation able to clean it when I need to.”
techniques.

D. Viral upper respiratory infections are a


common precipitating factor in acute asthma
attacks.

2. A patient is admitted to the emergency


department with an acute asthma attack.
Which patient assessment is of greatest
concern to the nurse?

A. The presence of a pulsus paradoxus

B. Markedly diminished breath sounds with no


wheezing
5. Which statements by a patient with
C. Use of accessory muscles of respiration and moderate asthma inform the nurse that the
a feeling of suffocation patient needs more teaching about
medications (Select All That Apply)?
D. A respiratory rate of 34 and increased pulse
and blood pressure A. “If I can’t afford all of my medicines, I will
only use the salmeterol (Serevent).”
3. Priority Decision: Which medication should
the nurse anticipate being used first in the B. “I will stay inside if there is a high pollen
emergency department for relief of severe count to prevent having an asthma attack.”
respiratory distress related to asthma?
C. “I will rinse my mouth after using
A. Prednisone orally fluticasone (Flovent HFA) to prevent oral
candidiasis.”
B. Ipratropium inhaler

C. Fluticasone inhaler
D. “I must have omalizumab (Xolair) injected equipment, what should the nurse teach the
every 2 to 4 weeks because inhalers don’t help patient?
my asthma.”
A. The portable unit will last about 6 to 8
E. “If I can’t afford all of my medicines, I will hours.
only use the salmeterol (Serevent).”
B. The unit is strictly for portable and
6. To decrease the patient’s sense of panic emergency use.
during an acute asthma attack, what is the
C. The unit concentrates O2 from the air,
best action for the nurse to do?
providing a continuous O2 supply.
A. Leave the patient alone to rest in a quiet,
D. Weekly delivery of one large cylinder of O2
calm environment.
will be necessary for a 7- to 10-day supply of
B. Stay with the patient and encourage slow, O2
pursed lip breathing.
9. During an acute exacerbation of mild COPD,
C. Reassure the patient that the attack can be the patient is severely short of breath and the
controlled with treatment. nurse identifies a nursing diagnosis of
ineffective breathing pattern related to
D. Let the patient know that frequent
obstruction of airflow and anxiety. What is the
monitoring is being done using measurement
best action by the
of vital signs and SpO2.
nurse?

7. When teaching the patient with mild asthma A. Prepare and administer routine
about the use of the peak flow meter, what bronchodilator medications.
should the nurse instruct the patient to do?
B. Perform chest physiotherapy to promote
A. Carry the flow meter with the patient at all removal of secretions.
times in case an asthma attack occurs
C. Administer oxygen at 5 L/min until the
B. Use the flow meter to check the status of shortness of breath is relieved.
the patient’s asthma every time the patient
D. Position the patient upright with the elbows
takes quick-relief medication
resting on the over-the-bed table
C. Follow the written asthma action plan (e.g.,
10. In planning care for the patient with
take quick-relief medication) if the expiratory
bronchiectasis, which nursing intervention
flow rate is in the yellow zone
should the nurse include?
D. Use the flow meter by emptying the lungs,
A. Relieve or reduce pain
closing the mouth around the mouthpiece, and
inhaling through the meter as quickly as B. Prevent paroxysmal coughing
possible
C. Prevent spread of the disease to others
8. A patient is being discharged with plans for
home O2 therapy provided by an O2 D. Promote drainage and removal of mucus
concentrator with an O2-conserving portable
unit. In preparing the patient to use the
SAS #6
1. To evaluate the effectiveness of prescribed a. The patient is somnolent.
therapies for a patient with ventilatory failure,
b. The patient's SpO2 is 90%.
which diagnostic test will be most useful to the
nurse? c. The patient complains of weakness.
a. Chest x-rays d. The patient's blood pressure is 162/94.
b. Pulse oximetry 5. A nurse answers a call light and finds a client
anxious, short of breath, reporting chest pain,
c. Arterial blood gas (ABG) analysis
and having a blood pressure of 88/52 mm Hg
d. Pulmonary artery pressure monitoring on the cardiac monitor. What action by the
nurse takes priority?
2. While caring for a patient who has been
admitted with a pulmonary embolism, the a. Assess the client’s lung sounds.
nurse notes a change in the patient's oxygen
b. Notify the Rapid Response Team.
saturation (SpO2) from 94% to 88%. The nurse
will c. Provide reassurance to the client.
a. increases the oxygen flow rate. d. Take a full set of vital signs.
b. suction the patient's oropharynx. 6. A client is admitted with a pulmonary
embolism (PE). The client is young, healthy,
c. assists the patient to cough and deep
and active and has no known risk factors for
breathe.
PE. What action by the nurse is most
d. helps the patient to sit in a more upright appropriate?
position.
a. Encourage the client to walk 5 minutes each
3. A patient with respiratory failure has a hour.
respiratory rate of 8 and an SpO2 of 89%. The
b. Refer the client to smoking cessation
patient is increasingly lethargic. The nurse will
classes.
anticipate assisting with
c. Teach the client about factor V Leiden
a. administration of 100% oxygen by non-
testing.
rebreather mask.
d. Tell the client that sometimes no cause for
b. endotracheal intubation and positive
disease is found.
pressure ventilation.
7. A client has a pulmonary embolism and is
c. insertion of a mini-tracheostomy with
started on oxygen. The student nurse asks why
frequent suctioning.
the clients oxygen saturation has not
d. initiation of bilevel positive pressure significantly improved. What response by the
ventilation (BiPAP). nurse is best?

4. When admitting a patient in possible a. Breathing so rapidly interferes with


respiratory failure with a high PaCO2, which oxygenation.
assessment information will be of most
b. Maybe the client has respiratory distress
concern to the nurse?
syndrome.
c. The blood clot interferes with perfusion in C. cardiomegaly and pulmonary fibrosis.
the lungs.
D. ventricular dysrhythmias and wheezing.
d. The client needs immediate intubation and
2. You are caring for a patient with a diagnosis
mechanical ventilation.
of iron-deficiency anemia. Which clinical
8. A client appears dyspneic, but the oxygen manifestations are you most likely to observe
saturation is 97%. What action by the nurse is when assessing this patient?
best?
A. Convex nails, bright red gums, and alopecia
a. Assess for other manifestations of hypoxia.
B. Brittle nails; smooth, shiny tongue; and
b. Change the sensor on the pulse oximeter. cheilosis

c. Obtain a new oximeter from central supply. C. Tenting of the skin, sunken eyes, and
complaints of diarrhea
d. Tell the client to take slow, deep breaths.
D. Pale pink tongue; dull, brittle hair; and blue
9. A nurse is assisting the health care provider
mucous membranes
who is intubating a client. The provider has
been attempting to intubate for 40 seconds. 3.When providing teaching for the patient with
What action by the nurse takes priority? iron-deficiency anemia who has been
prescribed iron supplements, you should
a. Ensure the client has adequate sedation.
include taking the iron with which beverage?
b. Find another provider to intubate.
A. Milk
c. Interrupt the procedure to give oxygen.
B. Ginger ale
d. Monitor the client’s oxygen saturation.
C. Orange juice
10. An intubated clients oxygen saturation has
D. Water
dropped to 88%. What action by the nurse
takes priority?

a. Determine if the tube is kinked.

b. Ensure all connections are patent. 4.The primary pathophysiology underlying


thalassemia is
c. Listen to the client’s lung sounds.
A. erythropoietin deficiency.
d. Suction the endotracheal tube.
B. abnormal hemoglobin synthesis.

C. autoimmunity.
SAS #7
D. S-shaped hemoglobin.
1. In a severely anemic patient, you expect to
find 5. Which individual is at high risk for a
cobalamin (vitamin B12) deficiency anemia?
A. dyspnea and tachycardia.
A. A 47-year-old man who had a gastrectomy
B. cyanosis and pulmonary edema.
(removal of the stomach)
B. A 54-year-old man with a history of irritable D. Strawberries
bowel disease and ulcerative colitis
E. Potatoes
C. A 26-year-old woman who complains of
10. You are evaluating the laboratory data of
heavy menstrual periods
the patient with suspected aplastic anemia.
D. A 15-year-old girl who is a vegetarian Which findings support this diagnosis?

6. You encourage the patient with cobalamin A. Reduced RBCs, reduced white blood cells
deficiency to seek treatment because (WBCs), and reduced platelets
untreated pernicious anemia may result in
B. Reduced RBCs, normal WBCs, and normal
A. death. platelets

B. liver failure. C. Normal RBCs, reduced WBCs, and reduced


platelets
C. heart failure.
D. Elevated RBCs, increased WBCs, and
D. gastrectomy.
increased platelets
7. The Schilling test for pernicious anemia
involves
SAS #8
A. administration of radioactive cobalamin and
measuring its excretion in the urine. 1. What are non-modifiable risk factors for
primary hypertension (select all that apply)?
B. blood cultures for organism identification.
a. Age
C. the measurement of serum iron.
b. Obesity
D. the administration of iron and blood
assessment of total iron binding in 24 hours. c. Gender

8. Which finding allows you to identify the d. Ethnicity


patient's anemia as folic acid deficiency rather
e. Genetic link
than cobalamin deficiency?
2. How is secondary hypertension
A. Loss of appetite
differentiated from primary hypertension?
B. Lack of neuromuscular symptoms
a. Has a more gradual onset than primary
C. Red tongue hypertension

D. Change in nail shape b. Does not cause the target organ damage
that occurs with primary hypertension
9. Which foods should you encourage patients
with folic acid deficiency to include in their c. Has a specific cause, such as renal disease,
daily food intake (select all that apply)? that often can be treated by medicine or
surgery
A. Ready-to-eat cereal
d. Is caused by age-related changes in BP
B. Wheat tortillas
regulatory mechanisms in people over 65 years
C. Lentils of age
3. What is the patient with primary d. “You need to remember that hypertension
hypertension likely to report? can be only controlled with medication, not
cured, and you must always take your
a. No symptoms
medication.”
b. Cardiac palpitations
6. A 78-year-old patient is admitted with a BP
c. Dyspnea on exertion of 180/98 mm Hg. Which age-related physical
changes may contribute to this patient’s
d. Dizziness and vertigo hypertension? (Select All That Apply)
4. A patient with stage 2 hypertension who is a. Decreased renal function
taking hydrochlorothiazide (Hydrodiuril) and
lisinopril (Prinivil) has prazosin (Minipress) b. Increased baroreceptor reflexes
added to the medication regimen. What is
c. Increased peripheral vascular resistance
most important for the nurse to teach the
patient to do? d. Increased adrenergic receptor sensitivity

a. Weigh every morning to monitor for fluid e. Increased collagen and stiffness of the
retention myocardium

b. Change position slowly and avoid prolonged f. Loss of elasticity in large arteries from
standing arteriosclerosis

c. Use sugarless gum or candy to help relieve 7. What should the nurse emphasize when
dry mouth teaching a patient who is newly prescribed
clonidine (Catapres)?
d. Take the pulse daily to note any slowing of
the heart rate a. The drug should never be stopped abruptly.

5. A 38-year-old man is treated for b. The drug should be taken early in the day to
hypertension with triamterene and prevent nocturia.
hydrochlorothiazide (Maxzide) and metoprolol
c. The first dose should be taken when the
(Lopressor). Four months after his last clinic patient is in bed for the night.
visit, his BP returns to pretreatment levels and
d. Because aspirin will decrease the drug’s
he admits he has not been taking his
effectiveness, Tylenol should be used instead.
medication regularly. What is the nurse’s best
response to this patient? 8. What is included in the correct technique for
BP measurements?
a. “Try always to take your medication when
you carry out another daily routine so you do a. Always take the BP in both arms.
not forget to take it.”
b. Position the patient supine for all readings.
b. “You probably would not need to take
medications for hypertension if you would c. Place the cuff loosely around the upper arm.
exercise more and stop smoking.” d. Take readings at least two times at least 1
c. “The drugs you are taking cause sexual minute apart
dysfunction in many patients. Are you
experiencing any problems? in this area?
9. Which manifestation is an indication that a d. Age 19, elevated low-density lipoprotein
patient is having a hypertensive emergency? (LDL) cholesterol, lipid-filled smooth muscle
cells
a. Symptoms of a stroke with an elevated BP
2. What accurately describes the
b. A systolic BP >200 mm Hg and a diastolic BP
pathophysiology of CAD?
>120 mm Hg
a. Partial or total occlusion of the coronary
c. A sudden rise in BP accompanied by
artery occurs during the stage of raised fibrous
neurologic impairment
plaque
d. A severe elevation of BP that occurs over
b. Endothelial alteration may be caused by
several days or weeks
chemical irritants such as hyperlipidemia or by
10. During treatment of a patient with a BP of tobacco use
222/148 mm Hg and confusion, nausea, and
c. Collateral circulation in the coronary
vomiting, the nurse initially titrates the
circulation is more likely to be present in the
medications to achieve which goal?
young patient with CAD
a. Decrease the mean arterial pressure (MAP)
d. The leading theory of atherogenesis
to 129 mm Hg
proposes that infection and fatty dietary
b. Lower the BP to the patient’s normal within intake are the basic underlying causes of
the second to third hour atherosclerosis

c. Decrease the SBP to 160 mm Hg and the DBP 3. While obtaining patient histories, which
to between 100- and 110-mm Hg as quickly as patient does the nurse identify as having the
possible highest risk for CAD?

d. Reduce the systolic BP (SBP) to 158 mm Hg a. A white man, age 54, who is a smoker and
and the diastolic BP (DBP) to 111 mm Hg has a stressful lifestyle
within the first 2 hours
b. A white woman, age 68, with a BP of
172/100 mm Hg and who is physically inactive

SAS #9 c. An Asian woman, age 45, with a cholesterol


level of 240 mg/dL and a BP of 130/74 mm Hg
1. Which patient is most likely to be in the
fibrous stage of development of coronary d. An obese African American man, age 65,
artery disease (CAD)? with a cholesterol level of 195 mg/dL and a BP
of 128/76 mm Hg
a. Age 40, thrombus adhered to the coronary
artery wall 4. Priority Decision: While teaching women
about the risks and incidence of CAD, what
b. Age 50, rapid onset of disease with does the nurse emphasize?
hypercholesterolemia
a. Smoking is not as significant a risk factor for
c. Age 32, thickened coronary arterial walls CAD in women as it is in men.
with narrowed vessel lumen
b. Women seek treatment sooner than men
when they have symptoms of CAD.
c. Estrogen replacement therapy in d. Angina in the patient with atherosclerotic
postmenopausal women decreases the risk for coronary arteries
CAD.
e. Left ventricular hypertrophy caused by
d. CAD is the leading cause of death in women, chronic hypertension
with a higher mortality rate after MI than in
f. Sympathetic nervous system stimulation by
men.
drugs, emotions, or exertion
5. Which characteristics are associated with
8. What causes the pain that occurs with
LDLs (select all that apply)?
myocardial ischemia?
a. Increases with exercise
a. Death of myocardial tissue
b. Contains the most cholesterol
b. Dysrhythmias caused by cellular irritability
c. Has an affinity for arterial walls
c. Lactic acid accumulation during anaerobic
d. Carries lipids away from arteries to liver metabolism

e. High levels correlate most closely with CAD d. Elevated pressure in the ventricles and
pulmonary vessels
6. Which serum lipid elevation, along with
elevated LDL, is strongly associated with CAD? 9. What types of angina can occur in the
absence of CAD (select all that apply)?
a. Apolipoproteins
a. Silent ischemia
b. Fasting triglycerides
b. Nocturnal angina
c. Total serum cholesterol
c. Prinzmetal’s angina
d. High-density lipoprotein (HDL)
d. Microvascular angina

e. Chronic stable angina

7. Myocardial ischemia occurs as a result of


increased oxygen demand and decreased 10. Which characteristics describe unstable
oxygen supply. What factors and disorders angina (select all that apply)?
result in increased oxygen demand (select all
a. Usually precipitated by exertion
that apply)?
b. Unpredictable and unrelieved by rest
a. Hypovolemia or anemia
c. Characterized by progressive severity
b. Increased cardiac workload with aortic
stenosis d. Occurs only when the person is recumbent
c. Narrowed coronary arteries from
atherosclerosis
SAS #10
1. Which of the following is the most common c. Myoglobin
symptom of myocardial infarction (MI)?
d. Troponin
a. Chest pain
6. A patient admitted to the hospital for
b. Dyspnea evaluation of chest pain has no abnormal
serum cardiac markers 4 hours after the onset
c. Edema
of pain. What noninvasive diagnostic test can
d. Palpitations be used to differentiate angina from other
types of chest pain?
2. An intravenous analgesic frequently
administered to relieve chest pain associated a. 12-lead ECG
with MI is:
b. Exercise stress test
a. Meperidine hydrochloride
c. Coronary angiogram
b. Hydromorphone hydrochloride
d. Transesophageal echocardiogram
c. Morphine sulfate
7. A 52-year-old man is admitted to the
d. Codeine sulfate emergency department with severe chest pain.
On what basis would the nurse suspect an MI?
3. The classic ECG changes that occur with an
MI include all of the following except: a. He has pale, cool, clammy skin.

a. An absent P wave b. He reports nausea and vomited once at


home.
b. An abnormal Q wave
c. He says he is anxious and has a feeling of
c. T-wave inversion impending doom.
d. ST segment elevation d. He reports he has had no relief of the pain
4. Which of the following statements about with rest or position change.
myocardial infarction pain is incorrect? 8. At what point in the healing process of the
a. It is relieved by rest and inactivity. myocardium following an infarct does early
scar tissue result in an unstable heart wall?
b. It is substernal in location.
a. 2 to 3 days after MI
c. It is sudden in onset and prolonged in
duration. b. 4 to 10 days after MI

d. It is viselike and radiates to the shoulders c. 10 to 14 days after MI


and arms. d. 6 weeks after MI
5. Myocardial cell damage can be reflected by 9. To detect and treat the most common
high levels of cardiac enzymes. The cardiac- complication of MI, what should the nurse do?
specific isoenzyme is:
a. Measure hourly urine output.
a. Alkaline phosphatase
b. Auscultate the chest for crackles.
b. Creatine kinase (CK-MB)
c. Use continuous cardiac monitoring. d. Decreased glomerular filtration rate and
edema
d. Take vital signs every 2 hours for the first 8
hours. 3. The acronym FACES is used to help educate
patients to identify symptoms of heart failure.
10. Which treatment is used first for the
What does this acronym mean?
patient with a confirmed MI to open the
blocked artery within 90 minutes of arrival to a. Frequent activity leads to cough in the
the facility? elderly and swelling

a. Stent placement b. Factors of risk: activity, cough, emotional


upsets, salt intake
b. Coronary artery bypass graft (CABG)
c. Follow activity plan, continue exercise, and
c. Percutaneous coronary intervention (PCI)
know signs of problems
d. Trans myocardial laser revascularization
d. Fatigue, limitation of activities, chest
(TMR)
congestion/cough, edema, shortness of breath

4. What is the pathophysiologic mechanism


SAS #11 that results in the pulmonary edema of left-
sided heart failure?
1. Which statements accurately describe heart
failure (select all that apply)? a. Increased right ventricular preload

a. A common cause of diastolic failure is left b. Increased pulmonary hydrostatic pressure


ventricular hypertrophy.
c. Impaired alveolar oxygen and carbon
b. A primary risk factor for heart failure is dioxide exchange
coronary artery disease (CAD).
d. Increased lymphatic flow of pulmonary
c. Systolic heart failure results in a normal left extravascular fluid
ventricular ejection fraction.

d. Systolic failure is characterized by abnormal


5. Which initial physical assessment finding
resistance to ventricular filling.
would the nurse expect to be present in a
e. Hypervolemia precipitates heart failure by patient with acute left-sided heart failure?
decreasing cardiac output and increasing
a. Bubbling crackles and tachycardia
oxygen consumption.
b. Hepatosplenomegaly and tachypnea
2. What describes the action of the natriuretic
peptides and nitric oxide in their counter c. Peripheral edema and cool, diaphoretic skin
regulatory processes in response to heart
d. Frothy blood-tinged sputum and distended
failure (HF)?
jugular veins
a. Excretion of potassium
6. A patient with chronic heart failure has
b. Increased release of ADH atrial fibrillation and a left ventricular ejection
fraction (LVEF) of 18%. To decrease the risk of
c. Vasodilation and decreased blood pressure
complications from these conditions, what received from the pharmacy, which medication
drug does the nurse anticipate giving? should not be included

a. Diuretics for this patient?

b. Anticoagulants a. Losartan (Cozaar)

c. β-Adrenergic blockers b. Carvedilol (Coreg)

d. Potassium supplements c. Dopamine (Intropin)

7. Which diagnostic test is most useful in d. Hydrochlorothiazide (HCTZ)


differentiating dyspnea related to pulmonary
effects of heart failure from dyspnea related to
pulmonary disease?

a. Exercise stress testing

b. Cardiac catheterization

c. B-type natriuretic peptide (BNP) levels

d. Determination of blood urea nitrogen (BUN)

8. Which medication is currently approved only


for use with African American patients for
hypertension and angina?

a. Captopril (Capoten)

b. Nitroglycerin (Nitro-Bid)

c. Spironolactone (Aldactone)

d. Isosorbide dinitrate and hydralazine (BiDil)

9. A patient is admitted to the emergency


department with acute decompensated heart
failure (ADHF). Which IV medication would the
nurse expect to administer first?

a. Digoxin (Lanoxin)

b. Morphine sulfate

c. Nesiritide (Natrecor)

d. Bumetanide (Bumex)

10. The patient with chronic heart failure is


being discharged with a diuretic, a renin-
angiotensin-aldosterone system (RAAS)
inhibitor and a β-adrenergic blocker. When
MEDSURG -1 P2 (SAS 12-22) d. Maintenance of normal body weight is the
most important factor in controlling arterial
disease.

SAS #12 5. During care of the patient following femoral


bypass graft surgery, the nurse immediately
notifies the health care provider if the patient
experiences
1. When obtaining a health history from a 72- a. fever and redness at the incision site.
year-old man with peripheral arterial disease b. 2+ edema of the extremity and pain at the
(PAD) of the lower extremities, the nurse asks incision site.
about a history of related conditions, including c. a loss of palpable pulses and numbness and
a. venous thrombosis. tingling of the feet.
b. venous stasis ulcers. d. increasing ankle-brachial indices and serous
c. pulmonary embolism. drainage from the incision.
d. coronary artery disease (CAD).
6. What are characteristic of arteriospastic
2. Following teaching about medications for disease (Raynaud’s phenomenon) (select all
PAD, the nurse determines that additional that apply)?
instruction is necessary when the patient
makes which statement? a. Predominant in young females
a. “I should take one aspirin a day to prevent b. May be associated with autoimmune
clotting in my legs.” disorders
b. “The lisinopril I use for my blood pressure c. Precipitated by exposure to cold, caffeine, and
may help me walk further without pain.” tobacco
c. “I will need to have frequent blood tests to d. Involves small cutaneous arteries of the
evaluate the effect of the Coumadin I will be fingers and toes
taking.” e. Inflammation of small and medium-sized
d. “Pletal should help me increase my walking arteries and veins
distance and help prevent clots from forming in f. Episodes involve white, blue, and red color
my legs.” changes of fingertips

3. A patient with PAD has a nursing diagnosis of 7. Which aneurysm is uniform in shape and a
ineffective peripheral tissue perfusion. What circumferential dilation of the artery?
should be included in the teaching plan for this a. False aneurysm
patient (select all that apply)? b. Pseudoaneurysm
a. Keep legs and feet warm. c. Saccular aneurysm
b. Apply cold compresses when the legs d. Fusiform aneurysm
become swollen.
c. Walk at least 30 minutes per day to the point 8. A surgical repair is planned for a patient who
of discomfort. has a 5.5-cm abdominal aortic aneurysm (AAA).
d. Use nicotine replacement therapy as a On physical assessment of the patient, what
substitute for smoking. should the nurse expect to find?
e. Inspect lower extremities for pulses, a. Hoarseness and dysphagia
temperature, and any injury. b. Severe back pain with flank ecchymosis
c. Presence of a bruit in the periumbilical area
4. When teaching the patient with PAD about d. Weakness in the lower extremities
modifying risk factors associated with the progressing to paraplegia
condition, what should the nurse emphasize?
a. Amputation is the ultimate outcome if the 9. A thoracic aortic aneurysm is found when a
patient does not alter lifestyle behaviors. patient has a routine chest x-ray. The nurse
b. Modifications will reduce the risk of other anticipates that additional diagnostic testing to
atherosclerotic conditions such as stroke. determine the size and structure of the
c. Risk-reducing behaviors initiated after aneurysm will include which test?
angioplasty can stop the progression of the
disease.
a. Angiography 5. The patient with acute pericarditis is having a
b. Ultrasonography pericardiocentesis. Postoperatively what
c. Echocardiography complication should the nurse monitor the
d. Computed tomography (CT) scan patient for?
a. Pneumonia
10. A patient with a small AAA is not a good b. Pneumothorax
surgical candidate. What should the nurse teach c. Myocardial infarction (MI)
the patient is one of the best ways to prevent d. Cerebrovascular accident (CVA)
expansion of the lesion?
a. Avoid strenuous physical exertion. 6. Priority Decision: A patient with acute
b. Control hypertension with prescribed pericarditis has a nursing diagnosis of pain
therapy. related to pericardial inflammation. What is the
c. Comply with prescribed anticoagulant best nursing intervention for the patient?
therapy. a. Administer opioids as prescribed on an
d. Maintain a low-calcium diet to prevent around-the-clock schedule.
calcification of the vessel. b. Promote progressive relaxation exercises with
the use of deep, slow breathing.
SAS #13 c. Position the patient on the right side with the
head of the bed elevated 15 degrees.
d. Position the patient in Fowler’s position with a
padded over-the-bed table for the patient to lean
1. A 20-year-old patient has acute infective on.
endocarditis. While obtaining a nursing history,
what should the nurse ask the patient about 7. When obtaining a nursing history for a patient
(select all that apply)? with myocarditis, what should the nurse
a. Renal dialysis specifically question the patient about?
b. IV drug abuse a. Prior use of digoxin for treatment of cardiac
c. Recent dental work problems
d. Cardiac catheterization b. Recent symptoms of a viral illness, such as
e. Recent urinary tract infection fever and malaise
c. A history of coronary artery disease (CAD)
2. A patient has an admitting diagnosis of acute with or without an MI
left-sided infective endocarditis. What is the best d. A recent streptococcal infection requiring
test to confirm this diagnosis? treatment with penicillin
a. Blood cultures
b. Complete blood count 8. Priority Decision: What is the most important
c. Cardiac catheterization role of the nurse in preventing rheumatic
d. Transesophageal echocardiogram fever? a. Teach patients with infective
endocarditis to adhere to antibiotic prophylaxis.
3. Which manifestation of infective endocarditis b. Identify patients with valvular heart disease
is a result of fragmentation and who are at risk for rheumatic fever.
microembolization of vegetative lesions? c. Encourage the use of antibiotics for treatment
of all infections involving a sore throat.
a. Petechiae d. Promote the early diagnosis and immediate
b. Roth’s spots treatment of group A streptococcal pharyngitis.
c. Osler’s nodes
d. Splinter hemorrhages 9. What manifestations most strongly support a
diagnosis of acute rheumatic fever?
4. What describes Janeway’s lesions that are a. Carditis, polyarthritis, and erythema
manifestations of infective endocarditis? marginatum
a. Hemorrhagic retinal lesions b. Polyarthritis, chorea, and decreased anti-
b. Black longitudinal streaks in nail beds streptolysin O titer
c. Painful red or purple lesions on fingers or c. Organic heart murmurs, fever, and elevated
toes erythrocyte sedimentation rate (ESR)
d. Flat, red, painless spots on the palms of d. Positive C-reactive protein, elevated white
hands and soles of feet blood cells (WBCs), and subcutaneous nodules
10. A patient with rheumatic heart disease with 15.A patient admitted with acute dyspnea is
carditis asks the nurse how long his activity will newly diagnosed with dilated cardiomyopathy.
be restricted. What is the best answer by the Which information will the nurse plan to teach
nurse? the patient about managing this disorder?
a. “Full activity will be allowed as soon as acute a. A heart transplant should be scheduled as
symptoms have subsided.” soon as possible.
b. “Bed rest will be continued until symptoms of b. Elevating the legs above the heart will help
heart failure are controlled.” relieve dyspnea.
c. “Nonstrenuous activities can be performed as c. Careful compliance with diet and medications
soon as antibiotics are started.” will prevent heart failure.
d. “Bed rest must be maintained until anti- d. Notify the doctor about any symptoms of heart
inflammatory therapy has been discontinued.” failure such as shortness of breath.

11. The patient is admitted post–radiation SAS #14


therapy with symptoms of cardiomyopathy
(CMP). Which type of CMP should the nurse
suspect that the patient is experiencing?
a. Dilated 1. When a burn damages cells, you would
b. Restrictive expect the cells to release the major
c. Takotsubo electrolyte:
d. Hypertrophic A. potassium.
B. chloride.
12. What accurately describes dilated CMP C. calcium.
(select all that apply)? D. sodium.
a. Characterized by ventricular stiffness
2. Diuretics affect the kidneys by altering the
b. The least common type of cardiomyopathy reabsorption and excretion of:
c. The hyperdynamic systolic function creates a A. water only.
diastolic failure B. electrolytes only.
d. Echocardiogram reveals cardiomegaly with C. water and electrolytes.
thin ventricular walls D. other drugs.
e. Often follows an infective myocarditis or
exposure to toxins or drugs 3. The main extracellular cation is:
f. Differs from chronic heart failure in that there
A. calcium.
is no ventricular hypertrophy
B. potassium.
C. bicarbonate.
13. When planning care for the patient with D. sodium.
hypertrophic CMP, what should the nurse
include? a. Ventricular pacing
4. Patient X is diagnosed with constipation. As a
b. Administration of vasodilators knowledgeable nurse, which nursing intervention
c. Teach the patient to avoid strenuous activity is appropriate for maintaining normal bowel
and dehydration function?
d. Surgery for cardiac transplantation will need A. Assessing dietary intake
to be done soon B. Decreasing fluid intake
14. When performing discharge teaching for a C. Providing limited physical activity
patient with any type of CMP, what should the D. Turning, coughing, and deep breathing
nurse instruct the patient to do (select all that
apply)? 5. A 12-year-old boy was admitted in the
a. Eat a low-sodium diet. hospital two days ago due to hyperthermia. His
b. Go to the gym every day. attending nurse, Dennis, is quite unsure about
c. Engage in stress reduction activities. his plan of care. Which of the following nursing
d. Abstain from alcohol and caffeine intake. intervention should be included in the care of
e. Avoid strenuous activity and allow for periods plan for the client?
of rest. A. Room temperature reduction
f. Suggest that caregivers learn cardiopulmonary B. Fluid restriction of 2,000 ml/day
resuscitation (CPR).
C. Axillary temperature measurements every 4 10. Which electrolyte would the nurse identify as
hours the major electrolyte responsible for determining
D. Antiemetic agent administration the concentration of the extracellular fluid?
A. Potassium
6. A 36-year-old male client is about to be B. Phosphate
discharged from the hospital after 5 days due to C. Chloride
surgery. Which intervention should be included D. Sodium
in the home health care nurse’s instructions
about measures to prevent constipation? A. SAS #15
Discouraging the client from eating large
amounts of roughage-containing foods in the
diet. B. Encouraging the client to use laxatives 1. Orly Khan is suffering from fluid
routinely to ensure adequate bowel elimination. volume deficit (FVD), which of the
C. Instructing the client to establish a bowel following symptoms would the nurse
evacuation schedule that changes every day. expect to assess in the patient?
D. Instructing the client to fill a 2-L bottle with A. rales
water every night and drink it the next day. B. bounding pulse
C. tachycardia
7. Mrs. Dela Riva is in her first trimester of D. bulging neck veins
pregnancy. She has been lying all day because
her OB-GYN requested her to have a complete 2. John Reid is admitted in the hospital
bed rest. Which nursing intervention is and is currently receiving hypertonic fluids.
appropriate when addressing the client’s need to Nursing management for the
maintain skin integrity? client includes monitoring for all of the
A. Monitoring intake and output accurately following potential complications except:
B. Instructing the client to cough and deep- A. water intoxication
breathe every 2 hours B. fluid volume excess (FVE)
C. Keeping the linens dry and wrinkle free C. cellular dehydration
D. Using a foot board to maintain correct D. cell shrinkage
anatomic position
3. Mr. Wenceslao is scheduled to receive an
8. A patient with tented skin turgor, dry mucous isotonic solution; which one of the following
membranes, and decreased urinary output is is an example of such solution?
under nurse Mark’s care. Which nursing
intervention should be included the care plan of A. D10% W
Mark for his patient? B. 0.45% saline
A. Administering I.V. and oral fluids C. 0.9% saline
B. Clustering necessary activities throughout the D. 3% normal saline
day
C. Assessing color, odor, and amount of 4. Which of the following statements
sputum provides the rationale for using a
D. Monitoring serum albumin and total protein hypotonic solution for a patient with
levels FVD?
9. A client with very dry mouth, skin and mucous A. A hypotonic solution provides
membranes is diagnosed of having dehydration. free water to help the kidneys
Which intervention should the nurse perform eliminate the solute.
when caring for a client diagnosed with fluid B. A hypotonic solution supplies an excess of
volume deficit? sodium and chloride ions.
A. Assessing urinary intake and output C. Excessive volumes are recommended in the
B. Obtaining the client’s weight weekly at early postoperative period.
different times of the day D. A hypotonic solution is used to treat
C. Monitoring arterial blood gas (ABG) results hyponatremia.
D. Maintaining I.V. therapy at the keep-vein-
open rate
5. The process of endocrine regulation of SAS #16
electrolytes involves:
A. sodium reabsorption and chloride excretion 1. A client's kidneys are retaining increased
B. chloride reabsorption and sodium excretion amounts of sodium. The nurse plans care,
C. potassium reabsorption and sodium anticipating that the kidneys also are
excretion retaining greater amounts of which
D. sodium reabsorption and potassium substances?
excretion A. Calcium and Chloride
B. Chloride and bicarbonate
6. Maria, an 85-year-old patient with a C. Potassium and Phosphates
feeding tube, has been experiencing severe D. Aluminum and magnesium
watery stool. The patient is lethargic and
has poor skin turgor, a pulse of 120, and 2. The nurse is preparing to care for a client
hyperactive reflexes. Nursing interventions with a potassium deficit. The nurse reviews the
would include: client's record and determines that the client
was at risk for developing the potassium deficit
A. measuring and recording intake and because of which situation?
output and daily weights A. Sustained tissue damage
B. administering salt tablets and monitoring B. Requires Nasogastric suction
hypertonic parenteral solutions C. Has a history of Addison's disease
C. administering sedatives D. Is taking a potassium-retaining diuretic
D. applying wrist restraints to avoid
displacement of the feeding tube 3. A nurse is planning care for a client
with hypokalemia. Which interventions
7. Disease of which of the following structures should be included in the plan of
is most likely to affect electrolyte reabsorption? care? Select all that apply.
A. Ensure adequate fluid intake.
A. glomerulus B. Implement safety measures to prevent falls
B. renal tubules C. Encourage low fiber foods to prevent
C. bladder diarrhea.
D. renal pelvis D. Instruct the client about foods that contain
potassium.
8. Daniel who is a marathon runner is at E. Encourage the client to obtain assistance to
high risk for fluid volume deficit. Which one ambulate.
of the following is a related factor? A.
decreased diuresis 4. The nurse is caring for a client with heart
B. disease-related process failure who is receiving high doses of a
C. decreased breathing and perspiration diuretic. On assessment, the nurse notes
D. increased breathing and perspiration that the client has flat neck veins, generalized
muscle weakness, and diminished deep
9. Jomarick is diagnosed with tendon reflexes. The nurse suspects
FVD; which of the following hyponatremia. What additional signs would
nursing diagnoses might apply the nurse expect to note in a client with
to his condition? A. altered hyponatremia? A. Muscle twitches
urinary elimination B. Decreased Urinary output
B. decreased cardiac output C. Hyperactive bowel sounds
C. increased cardiac output D. Increased specific gravity of the urine
D. vomiting
5. A patient prescribed spironolactone is
10. Body fluids perform which of the following demonstrating ECG changes &
functions? complaining of muscle weakness. The
A. transport nutrients nurse realizes this patient is exhibiting
B. transport electrical charges signs of which of the following.
C. cushion the organs A. hyperkalemia
D. facilitate fat metabolism B. hypokalemia
C. hypercalcemia SAS #17
D. hypocalcemia

6. The nurse is planning care for a patient 1. If your patient has hyperphosphatemia, he or
with fluid volume overload & hyponatremia. she may also have the secondary electrolyte
Which of the following should be included disturbance: A. hypermagnesemia.
in this patient's plan of care? B. hypocalcemia.
A. Restrict fluids. C. hypernatremia.
B. Administer intravenous fluids. D. hyperkalemia.
C. Provide Kayexalate.
D. Administer intravenous normal saline with 2. For a patient with hyperphosphatemia and
furosemide renal failure, avoid giving the phosphate binding
antacid:
7. A patient is prescribed 20 mEq of A. aluminum hydroxide.
potassium chloride. The nurse realizes that B. calcium carbonate.
the reason the patient is receiving C. calcium acetate.
this replacement is D. magnesium oxide.
A. to sustain respiratory function.
B. to help regulate acid-base balance. 3. Many of the signs and symptoms of
C. to keep a vein open. hypophosphatemia are related to:
D. to encourage urine output. A. low energy stores.
B. hypercalcemia.
8. A patient with fluid retention related to C. extensive diuresis.
renal problems is admitted to the hospital. D. hypocalcemia.
The nurse realizes that this patient could
possibly have which of the following 4. If your patient is hypercalcemic, you would
electrolyte imbalances? expect to:
A. hypokalemia A. administer I.V. sodium bicarbonate.
B. hypernatremia B. administer vitamin D.
C. carbon dioxide C. hydrate the patient.
D. magnesium D. administer digoxin.

9. An elderly patient comes into the clinic with 5. Hypercalcemia would be most likely to
the complaint of watery diarrhea for several develop in:
days with abdominal & muscle cramping. The A. a 60-year-old man who has squamous cell
nurse realizes that this pt is demonstrating carcinoma of the lung.
which of the following? B. an 80-year-old woman who has heart failure
A. hypernatremia and is taking furosemide (Lasix).
B. hyponatremia C. a 25-year-old trauma patient who has
C. fluid volume excess received massive blood transfusions.
D. hyperkalemia D. a 40-year-old man with hypoalbuminemia.

10. A patient is admitted with 6. You’re told during shift report that your patient
hypernatremia caused by being has a positive Chvostek’s sign. You would
stranded on a boat in the Atlantic Ocean expect his laboratory test results to reveal:
for five days without a fresh water A. a total serum calcium level below 8.9 mg/dl.
source. Which of the following is this B. a total serum calcium level above 10.1
patient at risk for developing? mg/dl.
A. pulmonary edema C. an ionized calcium level above 5.3 mg/dl.
B. atrial dysrhythmias D. an ionized calcium level between 4.4 and 5.3
C. cerebral bleeding mg/dl.
D. stress fractures
7. Your patient with Crohn’s disease develops a. Respiratory acidosis
tremors while receiving TPN. Suspecting she b. Respiratory alkalosis
might have hypomagnesemia; you assess her c. Metabolic acidosis
neuromuscular system. You should expect to d. Metabolic alkalosis
see:
A. Homans’ sign. 3. A patient who is lethargic and exhibits deep,
B. elevated serum potassium. rapid respirations has the following arterial blood
C. hyperactive DTRs. gas (ABG) results: pH 7.32, PaO2 88 mm Hg,
D. slowed heart rate PaCO2 37 mm Hg, and HCO3 16 mEq/L. How
should the nurse interpret these results? a.
8. When teaching your patient with Respiratory acidosis
hypomagnesemia about a proper diet, you b. Respiratory alkalosis
should recommend that he consume plenty of: c. Metabolic acidosis
A. seafood. d. Metabolic alkalosis
B. fruits.
C. corn products. 4.The nurse explains to a concerned family
D. dairy products. member of a client who has developed
respiratory acidosis that the kidneys a. achieve
optimal compensation immediately.
9. The doctor prescribes I.V. magnesium sulfate b. are unable to compensate.
for your patient with hypomagnesemia. Before c. can achieve optimal compensation in about 3
giving the magnesium preparation, you review days.
the practitioner’s order to make sure it specifies d. will compensate within 24 hours.
the:
A. number of grams or milliliters to give. 5.A client is admitted to the hospital with severe
B. number of ampules to give. vomiting and is diagnosed with metabolic
C. number of vials to give. alkalosis. The nurse anticipates that the
D. number of uses per vial. laboratory value that would support this
diagnosis
10. Your patient is diagnosed with a. arterial carbon dioxide tension (PaCO2) of 30
hypermagnesemia. To treat this imbalance, the mm Hg
practitioner is likely to order: A. magnesium b. arterial pH of 7.30.
citrate. c. serum calcium level of 9.0 mEq/L.
B. magnesium sulfate diluted in fluids. d. serum potassium level of 3.0 mEq/L
C. potassium-sparing diuretics.
D. oral and I.V. fluids. 6. A cigarette vendor was brought to the
emergency department of a hospital after she
SAS #18 fell into the ground and hurt her left leg. She is
noted to be tachycardic and tachypneic.
Painkillers were carried out to lessen her pain.
Suddenly, she started complaining that she is
1. A nurse assesses a client who is prescribed still in pain and now experiencing muscle
furosemide (Lasix) for hypertension. For which cramps, tingling, and
acid-base imbalance should the nurse assess paraesthesia. Measurement of arterial blood
to prevent complications of this therapy? gas reveals pH 7.6, PaO2 120 mm Hg, PaCO2
a. Respiratory acidosis 31 mm Hg, and HCO3 25 mmol/L. What does
b. Respiratory alkalosis this mean?
c. Metabolic acidosis a. Respiratory Alkalosis, Uncompensated
d. Metabolic alkalosis b. Respiratory Acidosis, Partially Compensated
c. Metabolic Alkalosis, Uncompensated
2. A patient who was involved in a motor vehicle d. Metabolic Alkalosis, Partially Compensated
crash has had a tracheostomy placed to allow
for continued mechanical ventilation. How 7. The nurse assesses that the client admitted in
should the nurse interpret the following arterial respiratory acidosis has compensated when the
blood gas results: pH 7.48, PaO2 85 mm Hg, arterial blood gas (ABG) readings are
PaCO2 32 mm Hg, and HCO3 25 mEq/L?
a. carbon dioxide level of 50 mm Hg and 2. When caring for a patient during the oliguric
bicarbonate level of 30 mEq/L. phase of acute kidney injury, what would be an
b. carbon dioxide level of 50 mm Hg and appropriate nursing intervention?
bicarbonate level of 20 mEq/L. A. Weigh patient three times weekly
c. carbon dioxide level of 30 mm Hg and B. Increase dietary sodium and potassium
bicarbonate level of 30 mEq/L. C. Provide a low-protein, high-carbohydrate
d. carbon dioxide level of 30 mm Hg and diet
bicarbonate level of 24 mEq/L. D. Restrict fluids according to the previous day's
fluid loss
8. For a 34-year-old client in renal failure who
develops acidosis, the nurse would assess for 3. Which assessment finding is commonly found
in the oliguric phase of acute kidney injury
a. drowsiness. (AKI)? A. Hypovolemia
b. hypoventilation. B. Hyperkalemia
c. muscle hyperactivity. C. Hypernatremia
d. paresthesia. D. Thrombocytopenia

9. Age-related physiologic changes the nurse 4. Which patient has the greatest risk for
would consider when planning care for an prerenal AKI?
elderly client admitted with an acid-base A. The patient is hypovolemic because of
abnormality include (Select all that apply) hemorrhage.
a. decreased pulmonary and renal function limit B. The patient relates a history of chronic urinary
the ability to compensate. tract obstruction.
b. hypermetabolism predisposes the elderly to C. The patient has vascular changes related to
metabolic acidosis. coagulopathies.
c. hypoventilation can easily cause respiratory D. The patient is receiving antibiotics such as
acidosis in the elderly. gentamicin.
d. renal perfusion is diminished because of
decreased cardiac output. 5. The patient admitted to the intensive care unit
e. there is decreased alveolar surface area for after a motor vehicle accident has been
gas exchange. diagnosed with AKI. Which finding indicates the
onset of oliguria resulting from AKI?
10. When evaluating the laboratory results of a A. Urine output less than 1000 mL for the past
patient with diabetic ketoacidosis, which lab 24 hours
value indicates the body has fully compensated B. Urine output less than 800 mL for the past 24
from this acid-base imbalance? hours
a. Normal serum glucose C. Urine output less than 600 mL for the past 24
b. Normal pH on arterial blood gases hours
c. Normal serum potassium D. Urine output less than 400 mL for the past 24
d. Normal bicarbonate on arterial blood gases hours

SAS #19 6. You are preparing to administer a dose of


PhosLo to a patient with chronic kidney disease
(CKD). This medication should have a
1. How do you determine that a patient's oliguria beneficial effect on which laboratory value?
is associated with acute renal failure (ARF)? A. Sodium
B. Potassium
A. Specific gravity of urine at 3 different times is C. Magnesium
1.010. D. Phosphorus
B. The serum creatinine level is normal.
C. The blood urea nitrogen (BUN) level is
normal or below.
D. Hypokalemia is identified.
7. A patient is admitted to the hospital with CKD. c. Malnutrition
You understand that this condition is
characterized by
d. Severe periodontal disease
A. Progressive irreversible destruction of the
kidneys 2. Which of the following
B. A rapid decrease in urinary output with an pathophysiological mechanisms that occurs
elevated BUN level in the lung parenchyma allows pneumonia
C. Increasing creatinine clearance with a
decrease in urinary output
to develop?
D. Prostration, somnolence, and confusion with
coma and imminent death a. Atelectasis
8. Nurses need to educate patients at risk for b. Bronchiectasis
CKD. Which individuals are considered to be at
increased risk (select all that apply)?
A. Older African Americans c. Effusion
B. Individuals older than 60 years
C. Those with a history of pancreatitis d. Inflammation
D. Those with a history of hypertension
E. Those with a history of type 2 diabetes
3. Which of the following organisms most
9. Patients with CKD have an increased commonly causes community-acquired
incidence of cardiovascular disease related to pneumonia in adults?
(select all that apply)
a. Hemophilus influenzae
A. hypertension.
B. vascular calcifications.
C. a genetic predisposition. b. Klebsiella pneumoniae
D. hyperinsulinemia causing dyslipidemia.
E. increased high-density lipoproteins levels. c. Streptococcus pneumoniae
10. Your plan for care of a patient with AKI
includes which goal of dietary management? d. Staphylococcus aureus
A. Provide sufficient calories while preventing
nitrogen excess. 4. An elderly client with pneumonia may
B. Deliver adequate calories while restricting fat appear with which of the following
and protein intake. symptoms first?
C. Replace protein intake with enough fat intake
to sustain metabolism.
D. Restrict fluids, increase potassium intake, a. Altered mental status and dehydration
and regulate sodium intake
b. Fever and chills
SAS #20
c. Hemoptysis and dyspnea

1. Clients with chronic illnesses are more d. Pleuritic chest pain and cough
likely to get pneumonia when which of the
following situations is present?

a. Dehydration

b. Group living
5. Following assessment of a patient with discharge instructions given by the nurse?
pneumonia, the nurse identifies a nursing
diagnosis of ineffective airwayclearance. a. "I will call the doctor if I still feel tired
Which information best supports this after a week."
diagnosis?
b. "I will need to use home oxygen therapy
a. Weak, nonproductive cough effort for 3 months."

b. Large amounts of greenish sputum c. "I will continue to do the deep breathing
and coughing exercises at home."
c. Respiratory rate of 28 breaths/minute
d. "I will schedule two appointments for the
d. Resting pulse oximetry (SpO2) of 85% pneumonia and influenza vaccines."

6. During assessment of the chest in a 9. Which nursing action will be most


patient with pneumococcal pneumonia, the effective in preventing aspiration
nurse would expect to find pneumonia in patients who are at risk?

a. vesicular breath sounds. a. Turn and reposition immobile patients at


least every 2 hours.
b. increased tactile fremitus.
b. Place patients with altered consciousness
c. dry, nonproductive cough. in side-lying positions.

d. hyperresonance to percussion. c. Monitor for respiratory symptoms in


patients who are immunosuppressed.
7. A patient with bacterial pneumonia has
rhonchi and thick sputum. Which action d. Provide for continuous subglottic
will the nurse use to promote airway aspiration in patients receiving enteral
clearance? feedings.

a. Assist the patient to splint the chest when 10. After a patient with right lower-lobe
coughing. pneumonia has been treated with
intravenous (IV) antibiotics for 2 days,
b. Educate the patient about the need for whichassessment data obtained by the
fluid restrictions. nurse indicates that the treatment has been
effective?
c. Encourage the patient to wear the nasal
oxygen cannula. a. Bronchial breath sounds are heard at the
right base.
d. Instruct the patient on the pursed lip
breathing technique. b. The patient coughs up small amounts of
green mucus.
8. Which statement by a patient who has
been hospitalized for pneumonia indicates a c. The patient's white blood cell (WBC)
good understanding of the count is 9000/μl.
d. Increased tactile fremitus is palpable about possible toxic effects of the antitubercular
over the right chest. medications, the nurse will give instructions to
notify the health care provider if the patient
develops
SAS #21
a. yellow-tinged skin.
b. changes in hearing.
c. orange-colored sputum.
d. thickening of the fingernails.
1. The health care provider writes an order for
bacteriologic testing for a patient who has a
6. An alcoholic and homeless patient is
positive tuberculosis skin test. Which action will
diagnosed with active tuberculosis (TB). Which
the nurse take?
intervention by the nurse will be most effective
a. Repeat the tuberculin skin testing.
in ensuring adherence with the treatment
b. Teach about the reason for the blood tests.
regimen?
c. Obtain consecutive sputum specimens from
a. Educating the patient about the long-term
the patient for 3 days.
impact of TB on health
d. Instruct the patient to expectorate three
b. Giving the patient written instructions about
specimens as soon as possible.
how to take the medications
c. Teaching the patient about the high risk for
2. Which information about a patient who has a
infecting others unless treatment is followed
recent history of tuberculosis (TB) indicates that
d. Arranging for a daily noontime meal at a
the nurse can discontinue airborne isolation
community center and giving the medication
precautions?
then
a. Chest x-ray shows no upper lobe infiltrates.
b. TB medications have been taken for 6
7. After 2 months of tuberculosis (TB) treatment
months.
with a standard four-drug regimen, a patient
c. Mantoux testing shows an induration of 10
continues to have positive sputum smears for
mm.
acid-fast bacilli (AFB). Which action should the
d. Three sputum smears for acid-fast bacilli are
nurse take next?
negative.
a. Ask the patient whether medications have
been taken as directed.
3. The nurse recognizes that the goals of
b. Discuss the need to use some different
teaching regarding the transmission of
medications to treat the TB.
pulmonary tuberculosis (TB) have been met
c. Schedule the patient for directly observed
when the patient with TB
therapy three times weekly.
a. demonstrates correct use of a nebulizer.
d. Educate about using a 2-drug regimen for the
b. washes dishes and personal items after use.
last 4 months of treatment.
c. covers the mouth and nose when coughing.
d. reports daily to the public health department.
8. A staff nurse has a tuberculosis (TB) skin test
of 16-mm induration. A chest radiograph is
4. Which information will the nurse include in the
negative, and the nurse has no symptoms of
patient teaching plan for a patient who is
TB. The occupational health nurse will plan on
receiving rifampin (Rifadin) for treatment of
teaching the staff nurse about the
tuberculosis?
a. "Your urine, sweat, and tears will be orange
a. use and side effects of isoniazid (INH).
colored."
b. standard four-drug therapy for TB.
b. "Read a newspaper daily to check for
c. need for annual repeat TB skin testing.
changes in vision."
d. bacilli Calmette-Guérin (BCG) vaccine.
c. "Take vitamin B6 daily to prevent peripheral
nerve damage."
9. When caring for a patient who is hospitalized
d. "Call the health care provider if you notice any
with active tuberculosis (TB), the nurse observes
hearing loss."
a family member who is visiting the patient. The
nurse will need to intervene if the family
5. When teaching the patient who is receiving
member
standard multidrug therapy for tuberculosis (TB)
a. washes the hands before entering the 2. Take off gown
patient's room. 3. Remove gloves
b. hands the patient a tissue from the box at the 4. Remove N95 respirator
bedside. 5. Perform hand hygiene
c. puts on a surgical face mask before visiting
the patient. a. 1, 2, 3, 4, 5
d. brings food from a "fast-food" restaurant to b. 2, 1, 4, 3, 5
the patient. c. 3, 4, 1, 2, 5
d. 4, 3, 2, 1, 5
10. Characteristics of the Mycobacterium
tuberculosis include all of the following except: 6. The nurse assessed the client and noted
a. It can be transmitted only by droplet nuclei. shortness of breath and recent trip to China. The
b. It is acid-fast. client is strongly suspected of having Middle
c. It is able to lie dormant within the body for East Respiratory Syndrome (MERS-CoV).
years. Which of these prescribed actions will the nurse
d. It survives in anaerobic conditions. take first?

SAS #22 a. Place the client on airborne and contact


precautions
b. Introduce normal saline at 75 mL/hr
1. Lassa and Ebola are emergent viruses in W. c. Give methylprednisolone (SOLU-Medrol) 1 g
Africa. What is their origin? intravenously (IV)
a. Humans d. Take blood, urine, sputum cultures
b. Primates
c. Fruit bats 7. The school nurse is asked which action will
d. Pigs take to have the most impact on the incidence of
infectious diseases in school. The correct
2. How can the morphology of filoviruses be response is:
described? a. Grant written information about infection
a. Very large ball like structure control to all parents
b. Long filamentous threads b. Ensure that students are immunized
c. Icosahedral virion according to national guidelines
d. Floppy membranes without defined c. Make soap and water easily accessible in the
morphology classrooms
d. Educate students on how to cover their
3. How can the spread of filoviruses be mouths when coughing
restricted?
a. New antivirals 8. A 10-year-old client contracted SARS when
b. New humanized monoclonal antibodies traveling abroad with her parents. The nurse
c. Hygiene and social distancing knows she must put on personal protective
d. New vaccine equipment to protect herself while providing
care. Based on the mode of SARS transmission,
4. What is the biggest risk factor for infection which personal protective should the nurse
with Ebola? wear?
a. Working in a category IV laboratory a. Gloves
b. Attending a funeral of a victim outside b. Gown and gloves
c. Nursing a patient at home c. Gown, gloves, and mask
d. Attending football matches or large d. Gown, gloves, mask, and eye goggles or eye
gatherings shield

5. In which order will the nurse perform the 9. A client who has recently traveled to Hong
following actions as she prepares to leave the Kong comes to the emergency department (ED)
room of a client with airborne precautions after with increasing shortness of breath and is
performing oral suctioning? strongly suspected of having a SARS. Which of
1. Take off goggles these prescribed actions will you take first?
a. Place the client on contact and airborne
precautions
b. Obtain blood, urine, and sputum for cultures
c. Administer methylprednisolone (Solu-Medrol)
1 gram/IV
d. Infuse normal saline at 100ml/hr

10. The school nurse is asked which action will


take to have the most impact on the incidence of
infectious diseases in school. The correct
response is:
a. Grant written information about infection
control to all parents
b. Ensure that students are immunized
according to national guidelines
c. Make soap and water easily accessible in the
classrooms
d. Educate students on how to cover their
mouths when coughing
MEDSURG -1 SAS 23-33 MULTIPLE CHOICE B. Use a condom during sexual intercourse

C. Have family members get an injection of


immunoglobin
SAS 23
D. Follow a low-protein, moderate-
1. The nurse questions a client with hepatitis B.
carbohydrate, moderate-fat diet
how long ago could the client have been
exposed to hepatitis B? 5. Which priority teaching information should
the nurse discuss with the client to help prevent
A. 3-7 days
contracting hep. B?
B. 7-14 days
A. Explain the importance of good hand
C. 40-50 days washing.

D. 60-160 days B. Tell the client to take the hepatitis B vaccine


in three (3) doses.
2. Which laboratory value increases would the
nurse expect to find in a client as a result of C. Tell the client not to ingest unsanitary food or
hepatitis? Select that all apply. water.

A. SGPT(ALT) and SGOT(AST) D. Discuss how to implement standard


precautions.
B. Alkaline Phosphatase
6. A client is suspected of having hepatitis.
C. bilirubin, ESR Which diagnostic test result will assist in
D. Leukocytes, Lymphocytes, Neutrophils confirming this diagnosis?

3. An adult client complains for frequent A. Elevated hemoglobin level


episodes of constipation. What is an effective B. Elevated serum bilirubin level
strategy for preventing
C. Elevated blood urea nitrogen level
constipation?
D. Decreased erythrocyte sedimentation rate
A. Reducing fluid intake to encourage bulk
formation in the intestinal lumen 7. A client with acute hepatitis is prescribed
lactulose. The nurse knows this medication will:
B. Use laxative daily to establish a regular
elimination pattern. A. Prevent the absorption of ammonia from the
bowel.
C. A regimen of exercise directed at toning the
abdominal muscles. B. Prevent hypoglycemia.

D. Setting a routine for bowel elimination just C. Remove bilirubin from the blood.
before bedtime
D. Mobilize iron stores from the liver
4. A patient with hepatitis B is being discharged
in 2 days. In the discharge teching plan the
nurse should include instructions to:

A. Avoid alcohol for the first 3 weeks


8. To prevent the spread of hepatitis A virus d. the importance of informing his partners of
(HAV) infection the nurse is especially careful the disease.
when:
2. Katrina Sterrett, a 26-year-old preschool
A. Disposing of food trays teacher, is being seen by a physician who is part
of the internist group where you practice
B. Emptying bed pans
nursing. She is undergoing her annual physical
C. Taking an oral temperature and is having many lab tests done as a condition
of her employment and upcoming wedding. She
D. Changing IV is returning for her results and is devastated to
9. Which type of hepatitis is transmitted by the learn that she has the sexuallytransmitted
fecal-oral route via contaminated food, water, infection, gonorrhea. What would contribute to
or direct contact with an infected person? her ignorance of her condition?

A. Hepatitis A a. Being asymptomatic

B. Hepatitis B b. All options are correct

C. Hepatitis C c. Being sexually inactive

D. Hepatitis D d. Knowing the signs and symptoms of STIs

10. The nurse is performing an assessment on a 3. Within the free clinic where you practice
client being evaluated for viral hepatitis. Which nursing, you hold weekly sexual education
symptom will the nurse most likely assess on classes open to the public. Within
this client? the classroom, you communicate the CDC's
A. Arthralgia numbers for the incidence of STIs and their
impact upon public health. Which
B. Excitability
is the fastest-spreading bacterial STI in the
C. Headache United States?
D. Polyphagia a. Gonorrhea
SAS 24 b. Chlamydia

c. Herpes simplex 1
1. A nurse is teaching a client with genital d. HPV
herpes. Education for this client should include
an explanation of: 4. A nurse is caring for a client diagnosed with a
chlamydia infection. The nurse teaches the
a. why the disease is transmittable only when client about disease transmission and advises
visible lesions are present. the client to inform his sexual partners of the
b. the need for the use of petroleum products. infection. The client refuses, stating, "This is my
business and I'm not telling anyone. Besides,
c. the option of disregarding safer-sex practices chlamydia doesn't cause any harm like the
now that he's already infected. other STDs." How should the nurse proceed?
a. Do nothing because the client's sexual habits 7. A patient comes to the clinic and requests
place him at risk for contracting other STDs. testing for HIV infection. Before administering
testing, what is most important for the nurse to
b. Educate the client about why it's important
do?
to inform sexual contacts so they can receive
treatment. a. Ask the patient to identify all sexual partners

c. Inform the health department that this client b. Determine when the patient thinks exposure
contracted an STD. to HIV occurred

d. Inform the client's sexual contacts of their c. Explain that all test results must be repeated
possible exposure to chlamydia. at least twice to be valid

c. Being sexually inactive

5. A 16-year-old patient comes to the free clinic d. Knowing the signs and symptoms of STIs
and is diagnosed with primary syphilis. The
d. Discuss prevention practices to prevent
patient states that she contracted this disease
transmission of the HIV to others
by holding hands with someone who has
syphilis. What is the most appropriate nursing 8. The “rapid” HIV antibody testing is performed
diagnosis for this patient? on a patient at high risk for HIV infection. What
should the nurse explain about this test?
a. Alteration in comfort related to impaired skin
integrity a. The test measures the activity of the HIV and
reports viral loads as real numbers.
b. Fear related to complications
b. This test is highly reliable, and in 5 minutes
c. Noncompliance with treatment regimen
the patient will know if HIV infection is present.
related to age
c. If the results are positive, another blood test
d. Knowledge deficit related to modes of
and a return appointment for results will be
transmission
necessary.
6. Which opportunistic disease associated with
d. This test detects drug-resistant viral
AIDS is characterized by hyperpigmented
mutations that are present in viral genes to
lesions of skin, lungs, and gastrointestinal (GI)
evaluate resistance to antiretroviral drugs.
tract?
9. Treatment with two nucleoside reverse
a. Kaposi sarcoma
transcriptase inhibitors (NRTIs) and a protease
b. Herpes simplex type 1 infection inhibitor (PI) is prescribed for a patient with HIV
infection who has a CD4+ T-cell count of
c. Candida albicans
<400/μL. The patient asks why so many drugs
d. Varicella-zoster virus infection are necessary for treatment. What should the
nurse explain as the primary rationale for
combination therapy?

a. Cross-resistance between specific


antiretroviral drugs is reduced when drugs are
given in combination.
b. Combinations of antiretroviral drugs lower quadrant. The nurse will document this as
decrease the potential for development of which of the following diagnostic signs of
antiretroviral-resistant HIVvariants. appendicitis?

c. Side effects of the drugs are reduced when a. Rovsing sign


smaller doses of three different drugs are used
b. referred pain
rather than large doses of one drug.
c. Chvostek's sign
d. When CD4+ T-cell counts are <500/μL, a
combination of drugs that have different d. rebound tenderness
actions is more effective in slowing HIV growth.
3. Which of the following position should the
10. What is one of the most significant factors client with appendicitis assume to relieve pain?
in determining when to start antiretroviral
therapy in a patient with HIV infection? a. Prone

a. Whether the patient has high levels of HIV b. Sitting


antibodies c. Supine
b. Confirmation that the patient has contracted d. Lying with legs drawn up
HIV infection

c. The patient’s readiness to commit to a


complex, lifelong, uncomfortable drug regimen 4. When evaluating a male client for
complications of acute pancreatitis, the nurse
d. Whether the patient has a support system to would observe for:
help manage the costs and side effects of the
drugs a. increased intracranial pressure.

SAS 25 b. decreased urine output.

1. The nurse would increase the comfort of the c. bradycardia.


patient with appendicitis by: d. hypertension.
a. Having the patient lie prone 5. When preparing a male client, age 51, for
b. Flexing the patient's right knee surgery to treat appendicitis, the nurse
formulates a nursing diagnosis of Risk for
c. Sitting the patient upright in a chair infection related to inflammation, perforation,
d. Turning the patient onto his or her left side and surgery. What is the rationale for choosing
this nursing diagnosis?
2. The nurse is caring for a patient in the
emergency department with complaints of a. Obstruction of the appendix may increase
acute abdominal pain, nausea, and vomiting. venous drainage and cause the appendix to
When the nurse palpates the patient's left rupture.
lower abdominal quadrant, the patient b. Obstruction of the appendix reduces arterial
complains of pain in the right flow, leading to ischemia, inflammation, and
rupture of the appendix.
c. The appendix may develop gangrene and 9. Which of the following conditions is most
rupture, especially in a middle-aged client. likely to directly cause peritonitis?

d. Infection of the appendix diminishes necrotic a. Cholelithiasis


arterial blood flow and increases venous
b. Gastritis
drainage.
c. Perforated ulcer

d. Incarcerated hernia
6. A client with diverticulitis has developed
peritonitis following diverticular rupture. The 10. Which of the following symptoms would a
nurse should assess the client to determine client in the early stages of peritonitis exhibit?
which of the following? Select all that apply.
a. Abdominal distention
a. Percuss the abdomen to note resonance and
tympany. b. Abdominal pain and rigidity

b. Percuss the liver to note lack of dullness. c. Hyperactive bowel sounds

c. Monitor the vital signs for fever, tachypnea, d. Right upper quadrant pain
and bradycardia. SAS 26
d. Assess presence of polyphagia and 1. A nurse is completing the admission
polydipsia. assessment of a client who has acute
e. Auscultate bowel sounds to note frequency. pancreatitis. Which finding is the first priority?

7. A client with acute appendicitis develops a A. History of cholelithiasis


fever, tachycardia, and hypotension. Based on B. Elevated serum amylase levels
these assessment findings, the nurse should
further assess the client for which of the C. Decrease in bowel sounds upon auscultation
following complications? D. Hand spasms present when blood pressure is
a. Deficient fluid volume. checked

b. Intestinal obstruction. 2. A nurse is completing an admission


assessment of a client who has pancreatitis.
c. Bowel ischemia. Which of the following findings should the
d. Peritonitis. nurse expect?

8. A client has just had surgery for colon cancer. A. Pain in the UQ rating to the shoulder
Which of the following disorders might the B. Report of pain being worse when sitting
client develop? upright
a. Peritonitis C. Pain relieved with defecation
b. Diverticulosis D. Epigastric pain radiating to the left shoulder
c. Partial bowel obstruction

d. Complete bowel obstruction


3. A nurse is assessing a client who has 6. Nursing management of the patient with
pancreatitis. Which of the following actions acute pancreatitis includes: (SATA)
should the nurse take to assess the presence of
A. Check for signs of hypocalcemia
Cullen's sign?
B. Provide a diet low in carbohydrates
A. Tap lightly at the costovertebral margin on
the client's back C. Giving insulin based on sliding scale
B. Palpate the RLQ D. Observing stools for signs of steatorrhea
C. Inspect the skin around the umbilicus E. Monitoring for infection, particularly
respiratory tract infection
D. Auscultate the area below the scapula

7. A patient with sudden pain in the left upper


4. A nurse is completing nutritional teaching for
quadrant radiating to the back and vomiting
a client who has pancreatitis. Which of the
was diagnosed with acute pancreatitis. Which
following statements by the client indicates an
intervention should the nurse include in the
understanding of the teaching? (SATA)
patient's plan of care?
A. I plan to eat small, frequent meals.
A. Immediately start enteral feeding to prevent
B. I will eat easy-to-digest foods with limited malnutrition.
spice
B. Insert an NG and maintain NPO status to
C. I will use skim milk when cooking allow pancreas to rest.

D. I plan to drink regular cola C. Initiate early prophylactic antibiotic therapy


to prevent infection.
E. I will limit alcohol intake to two drinkers per
day D. Administer acetaminophen (Tylenol) every 4
hours for pain relief.
5. The nurse is caring for a 55-yr-old man
patient with acute pancreatitis resulting from 8. The client is admitted to the medical
gallstones. Which clinical manifestation would department with a diagnosis of R/O acute
the nurse expect? pancreatitis. What laboratory values should the
nurse monitor to confirm this diagnosis?
A. Hematochezia
A. Creatinine and BUN
B. Left upper abdominal pain
B. Troponin and CK-MB
C. Ascites and peripheral edema
C. Serum amylase and lipase
D. Temperature over 102 F
D. Serum bilirubin and calcium

9. What client problem has a priority for the


client diagnosed with acute pancreatitis?

A. Risk for fluid volume deficient

B. Alteration in comfort
C. Imbalanced nutrition: less than the boy C. Hereditary
requires
D. Lack of exercise
D. Knowledge deficient

10. The client is diagnosed with acute


4. Which of the following factors is believed to
pancreatitis. What health-care provider's
cause ulcerative colitis?
admitting order should the nurse question?
A. Acidic diet
A. Bedrest with bathroom privileges
B. Altered immunity
B. Initiate IV therapy of D5W at 125 mL/hr.
C. Chronic constipation
C. Weight client daily
D. Emotional stress
D. Low fat, low carb diet
5. Fistulas are most common with which of the
following bowel disorders?
SAS 27
A. Crohn's disease
1. Crohn's disease can be described as a chronic
B. Diverticulitis
relapsing disease. Which of the following areas
in the GI system may be involved with this C. Diverticulosis
disease?
D. Ulcerative colitis
A. The entire length of the large colon
6. Which of the following areas is the most
B. Only the sigmoid area common site of fistulas in clients with Crohn's
disease?
C. The entire large colon through the layers of
mucosa and submucosa A. Anorectal
D. The small intestine and colon; affecting the B. Ileum
entire thickness of the bowel
C. Rectovaginal
2. Which area of the alimentary canal is the
most common location for Crohn's disease? D. Transverse colon

A. Ascending colon the intestinal wall in this area.

B. Descending colon 7. Which of the following associated disorders


may a client with ulcerative colitis exhibit?
C. Sigmoid colon
A. Gallstones
D. Terminal ileum
B. Hydronephrosis
3. Which of the following factors is believed to
be linked to Crohn's disease? C. Nephrolithiasis

A. Constipation D. Toxic megacolon

B. Diet
8. Which of the following associated disorders 2. A client scheduled for a cholecystectomy asks
may the client with Crohn's disease exhibit? what caused the gallstones to develop. Which
risk factor should the nurse list when
A. Ankylosing spondylitis
responding to this client? (Select all that apply.)
B. Colon cancer
A. American Indian ethnicity
C. Malabsorption
B. Male sex
D. Lactase deficiency
C. Family history of gallstones
9. Which of the following symptoms is
D. Obesity
associated with ulcerative colitis?
E. Hyperlipidemia
A. Dumping syndrome
3. A client asks what causes gallstones to form.
B. Rectal bleeding
Which factor should the nurse explain as being
C. Soft stools present when these stones are formed? (Select
all that apply.)
D. Fistulas
A. Rapid weight gain
10. If a client had irritable bowel syndrome,
which of the following diagnostic tests would B. Abnormal bile composition
determine if the diagnosis is Crohn's disease or
C. Excess cholesterol
ulcerative colitis?
D. Inflammation of the gallbladder
A. Abdominal computed tomography (CT) scan
E. Biliary stasis
B. Abdominal x-ray
4. A client with acute cholecystitis is
C. Barium swallow
experiencing jaundice. Which should the nurse
D. Colonoscopy with biopsy consider as the reason for the jaundice?

A. Viral infection of the gallbladder

B. Obstruction of the cystic duct by a gallstone

SAS 28 C. Accumulation of bile in the hepatic duct

1. Which is a risk factor for gallbladder disease? D. Accumulation of fat in the wall of the
gallbladder
A. Male gender
5. A 40-year-old client is admitted to the
B. Hypocalcemia hospital with cholecystitis. The nurse should
C. Rapid weight loss contact the physician to question which of the
following prescriptions?
D. Hypolipidemia
A. IV fluid therapy of normal saline solution to
be infused at 100 mL/h until further
prescriptions
B. Administer morphine sulfate 10 mg IM every A. Counseling regarding low-fat menu choices
4 hours as needed for severe abdominal pain
B. Administering antiemetics as prescribed
C. Nothing by mouth (NPO) until further
C. Assessing height and weight
prescriptions
D. Advising to consume a low-protein diet
D. Insert a nasogastric tube and connect to low
intermittent suction E. Reviewing serum electrolytes
6. The nurse is preparing health promotion 9. A client is recovering from a laparoscopic
teaching for a client with gallbladder disease. cholecystectomy. Which nursing action should
Which topic should the nurse include in the the nurse use to reduce this client's risk of
teaching session? (Select all that apply.) infection? (Select all that apply.)
A. Role of a high-cholesterol diet on gallstone A. Monitor vital signs, including temperature,
formation every 4 hours.
B. Role of hypolipidemia on gallstone formation B. Administer antibiotics as prescribed.
C. Importance of a low-cholesterol diet C. Coach to take deep breaths every 1dash2
hours while awake.
D. Dangers of rapid weight loss
D. Assess the abdomen every 4 hours.
E. Importance of a high-fiber diet
E. Place in Fowler position.
7. The nurse prepares discharge teaching for a
client recovering from a cholecystectomy. 10. The nurse evaluates a client's understanding
Which topic should the nurse include in this of discharge teaching following a laparoscopic
teaching? (Select all that apply.) cholecystectomy. Which client statement
indicates teaching has been effective? (Select all
A. Surgical incision care
that apply.)
B. Manifestations of postoperative
A. "I will take my pain medicine on an empty
complications
stomach to get the maximum benefit."
C. Pain control measures
B. "I will be sure to get up and walk every hour."
D. Activity level
C. "I can have some hot chocolate with my
E. High-fat diet breakfast."

8. A client with acute cholecystitis is D. "I will increase the protein in my diet by
experiencing nausea and vomiting. Which drinking whole milk."
nursing action should the nurse use to address
this client's nutritional status? (Select all that
apply.)
SAS 29 4. An older male patient states that he is having
problems starting and stopping his stream of
1. When caring for the patient with interstitial
urine and he feels the urgency to void. The best
cystitis, what can the nurse teach the patient to
way to assist this patient is to
do?
A. Help him stand to void.
A. Avoid foods that make the urine more
alkaline. B. Place a condom catheter.

B. Use high-potency vitamin therapy to C. Have him practice Credé's method.


decrease the autoimmune effects of the
D. Initiate Kegel exercises.
disorder.
5. A 24-year-old female client comes to an
C. Always keep a voiding diary to document
ambulatory care clinic in moderate distress with
pain, voiding frequency, and patterns of
a probable diagnosis of acute cystitis. When
nocturia.
obtaining the client's history, the nurse should
D. Use the dietary supplement calcium ask the client if she has had:
glycerophosphate (Prelief) to decrease bladder
A. Fever and chills.
irritation.
B. Frequency and burning on urination.
2. A female patient reports that she is
experiencing burning on urination, frequency, C. Flank pain and nausea.
and urgency. The nurse notes that a clean-
voided urine specimen is markedly cloudy. The D. Hematuria.
probable cause of these symptoms and findings 6. A client who has been diagnosed with renal
is: calculi reports that the pain is intermittent and
A. Cystitis. less colicky. Which of the following nursing
actions is most important at this time?
B. Hematuria.
A. Report hematuria to the physician.
C. Pyelonephritis.
B. Strain the urine carefully.
D. Dysuria.
C. Administer meperidine (Demerol) every 3
3. In cystitis to minimize experiencing nocturia, hours.
the nurse would teach the patient to:
D. Apply warm compresses to the flank area.
A. Perform perineal hygiene after urinating.
7. The client is scheduled for an intravenous
B. Set up a toileting schedule. pyelogram (IVP) to determine the location of
the renal calculi. Which of the following
C. Double void.
measures would be most important for the
D. Limit fluids before bedtime. nurse to include in pretest preparation?

A. Ensuring adequate fluid intake on the day of


the test.

B. Preparing the client for the possibility of


bladder spasms during the test.
C. Checking the client's history for allergy to SAS 30
iodine.
1. A 16-year-old sexually active female patient
D. Determining when the client last had a bowel with a history of pelvic inflammatory disease
movement. (PID) presents to the emergency room with
complaints of sudden right-sided lower
8. After an intravenous pyelogram (IVP), the
abdominal pain and gastrointestinal distress.
nurse should anticipate incorporating which of
She cannot recall the date of her last menstrual
the following measures into
period, but states she knows that she’s “late.”
the client's plan of care? Her exam demonstrates a unilateral, right
adnexal mass. The nurse knows that this patient
A. Maintaining bed rest. likely has which of these prenatal
B. Encouraging adequate fluid intake. complications?

C. Assessing for hematuria. A. gestational trophoblastic disease (GTD)

D. Administering a laxative. B. spontaneous abortion

9. Because a client's renal stone was found to C. ectopic pregnancy


be composed of uric acid, a low-purine, D. premature rupture of membranes (PROM)
alkaline-ash diet was ordered. Incorporation of
which of the following food items into the 2. Which statement by the nurse demonstrates
home diet would indicate that the client effective communication techniques when
understands the necessary diet modifications? initiating a discussion about sex with a 25-year-
old female client?
A. Milk, apples, tomatoes, and corn.
A. "Do you know how to properly apply a male
B. Eggs, spinach, dried peas, and gravy. condom?"
C. Salmon, chicken, caviar, and asparagus. B. "What questions do you have related to your
D. Grapes, corn, cereals, and liver. sexual health?"

10. Allopurinol (Zyloprim), 200 mg/ day, is C. "Have you had sex with more than one
prescribed for the client with renal calculi to partner?"
take at home. The nurse should teach the client D. "Why didn't you start receiving annual Pap
about which of the following adverse effects of tests at an earlier age?"
this medication?
3. While caring for a client who is being treated
A. Retinopathy. for severe pelvic inflammatory disease (PID),
B. Maculopapular rash. the nurse insists on keeping her in a semi sitting
position. What would be the best possible
C. Nasal congestion. reason for the nurse's advice?
D. Dizziness. A. To prevent nosocomial infections to other
clients

B. To facilitate easy distraction of the client

C. To prevent movement as it increases pain


D. To facilitate pelvic drainage and to minimize 7. A man whose BPH has been successfully
the upward extension of infection managed through medical treatment visits the
provider's office and reports he has suddenly
4. While caring for a patient who is being
had a return of symptoms including frequency,
treated for severe pelvic inflammatory disease
urgency, and a sensation of incomplete
(PID), which of the following nursing actions
emptying after voiding. The nurse collects a
minimizes transmission of infection?
thorough history and suspects the possible
A. Keeping the patient in a sitting position cause of the sudden exacerbation of the client's
symptoms may be:
B. Performing hand hygiene when entering the
room A. increased sexual activity since his wife has
retired.
C. Strictly adhering to the no visitation policy
B. antihypertensive medications he was
D. Implementing reverse isolation precautions recently prescribed.
5. The nurse is obtaining the history from a C. increased levels of exercise as he trains for a
client who is suspected of having pelvic marathon.
inflammatory disease (PID). Which client
statement would help support the suspicion of D. over-the-counter medications he's been
PID? taking to treat cold symptoms.

A. "I haven't had sex with anyone else except 8. The nurse recognizes that urinary elimination
my current partner." changes may occur even in healthy elders
because:
B. "My partner and I use condoms during sexual
intercourse." A. the bladder distends, and its capacity
increases.
C. "I was 15 years old when I first had sex."
B. elders ignore the need to void.
D. "I've never had any sexually transmitted
infection." C. the amount of urine retained after voiding
increases.
6. The nurse is assessing a client who is
suspected of experiencing an enlarging prostate D. urine becomes more concentrated.
gland (BPH). The nurse expects the enlarging
9. The nurse is taking the history of a client who
prostate in BPH to be manifested by which of
has had benign prostatic hyperplasia in the
the following symptoms?
past. To determine whether the client currently
A. Bowel elimination is experiencing difficulty, the nurse asks the
client about the presence of which of the
B. Skin integrity following early symptoms?
C. Peripheral vascular function A. Urge incontinence
D. Urinary elimination B. Nocturia

C. Decreased force in the stream of urine

D. Urinary retention
10. The client asks, "What does an elevated PSA D) "Fever is a known trigger for an SLE
test mean?" On which scientific rationale exacerbation."
should the nurse base the response?
3) The nurse is providing health education to a
A. An elevated PSA can result from several diverse group at a neighborhood community
different causes. center. Why does the nurse plan to include
signs and symptoms of systemic lupus
B. An elevated PSA can be only from prostate
erythematosus (SLE)?
cancer.
A) The neighborhood is composed of many
C. An elevated PSA can be diagnostic for
young female children.
testicular cancer.
B) The audience has asked the nurse to include
D. An elevated PSA is the only test used to
the information.
diagnose BPH.
C) The audience is mainly composed of
SAS 31
Caucasian women.
1) The client enters the outpatient clinic and
D) The audience is mainly females of Asian-
states to the triage nurse, "I think I have the flu.
American descent.
I'm so tired, I have no appetite, and everything
hurts." The triage nurse assesses the client and 4) The nurse is caring for a client who is
finds a butterfly rash over the bridge of nose hospitalized due to an exacerbation of systemic
and on the cheeks. Which diagnosis does the lupus erythematosus (SLE). The nurse is
nurse expect? reviewing the client's lab work and finds the
white blood cell count (WBC) is shifted to the
A) Systemic lupus erythematosus
left. Based on this information, which is a
B) Fibromyalgia priority nursing diagnosis for this client?

C) Lyme disease A) Ineffective Protection

D) Gout B) Ineffective Health Maintenance

2) A female client asks the nurse if there are any C) Ineffective Individual Coping
conditions that can exacerbate systemic lupus
D) Risk for Impaired Skin Integrity
erythematosus (SLE). Which is the best nurse
response? 5) A client with SLE is being treated with
immunosuppressant drugs and corticosteroids.
A) "Conditions that cause hypotension can
Which precautions should the nurse provide
often exacerbate SLE."
this client? Select all that apply.
B) "GI upset is often associated with SLE
A) Avoid large crowds.
exacerbation."
B) Don't get a flu shot.
C) "Pregnancy is often associated with an SLE
exacerbation." C) Use contraception to prevent pregnancy

D) Refrain from taking aspirin or ibuprofen.

E) Report signs of infection to the physician.


6) A nurse is caring for a client with systemic 9) The nurse is providing care for a newly
lupus erythematosus (SLE). The client begins to married woman with systemic lupus
cry and tells the nurse that she is afraid that her erythematosus (SLE). Which client statement
skin will be disfigured with lesions. Which indicates plan of care understanding?
intervention does the nurse plan to teach this
A) "I will take birth control pills while I am
client to minimize skin infections associated
taking cytotoxic medications."
with SLE? Select all that apply.
B) "I do not need to contact the doctor if I
A) Use sunscreen with an SPF of 15 or greater.
develop a fever or rash."
B) Remain indoors on sunny days.
C) "I plan to go to the movies this weekend so
C) Avoid swimming in a pool or the ocean. that I get out of the house."

D) Avoid sun exposure between 10:00 a.m. and D) "I can take ibuprofen as indicated for pain."
3:00 p.m.
10) A nurse is caring for a client with systemic
E) Decrease sun exposure between 3:00 p.m. lupus erythematous (SLE) who is taking
and 5:00 p.m. hydroxychloroquine (Plaquenil). The nurse
understands that the primary concern with this
7) The nurse is caring for a client who has been
drug is:
diagnosed with discoid lupus erythematosus.
The nurse is collaborating with the client to set A) Pulmonary fibrosis.
goals for the nursing plan of care. What is an
B) Cushingoid effects.
appropriate goal for this client?
C) Retinal toxicity.
A) Work through the stages of death and dying.
D) Renal toxicity
B) Comply 100% of the time with a sun
protection plan.

C) Gain weight to within 10 pounds of normal SAS 32


for height.
1. When administering a blood transfusion to a
D) Report pain no higher than four on a scale of client with multiple traumatic injuries, the nurse
1-10. monitors closely for evidence of a transfusion
reaction. Shortly after the transfusion begins,
8) The nurse is planning care for an adolescent
the client complains of chest pain, nausea, and
client who has systemic lupus erythematosus
itching and there is a rise in the client's
(SLE). The nurse knows that the treatment plan
temperature. The nurse stops the transfusion
implemented by the healthcare team is
and notifies the physician. The nurse suspects
appropriate for the situation when the client:
which type of hypersensitivity reaction with a
A) Refuses to attend school. blood transfusion?

B) Does not want to attend any social functions. a) Type II (cytolytic, cytotoxic) hypersensitivity
reaction
C) Discusses skin changes with the healthcare
personnel. b) Type IV (cell-mediated, delayed)
hypersensitivity reaction
D) Discusses skin changes with a good friend.
c) Type I (immediate, anaphylactic) d) Fruit salad and mineral water
hypersensitivity reaction
5. A client develops a facial rash and urticaria
d) Type III (immune complex) hypersensitivity after receiving penicillin. Which laboratory
reaction value does the nurse expect to be elevated?

2. A nurse practicing in a nurse-managed clinic a) IgE


suspects that an 8-year-old child's chronic
b) IgG
sinusitis and upper respiratory tract infections
may result from allergies. Which laboratory test c) IgA
would the nurse most likely order? Select all
that apply. d) IgB

a) Metabolic panel 6. A nurse practicing in a nurse-managed clinic


suspects that an 8-year-old child's chronic
b) Rheumatoid factor sinusitis and upper respiratory tract infections
may result from allergies. She orders an
c) Immunoglobulin assay (IgE)
immunoglobulin assay. Which immunoglobulin
d) Liver function studies would the nurse expect to find elevated?

e) Complete blood count a) Immunoglobulin M

3. During a school party a child with a known b) Immunoglobulin E


food allergy has an itchy throat, is wheezing,
c) Immunoglobulin D
and reports not feeling "quite right." The nurse
should do the following in what order from first d) Immunoglobulin G
to last? All options must be used.

a) Assess vital signs.


7. A nurse is caring for a client with the
b) Position to facilitate breathing. following laboratory values: white blood cell
count (WBC) 4,500/mm3, neutrophils 15%, and
c) Send someone to activate the Emergency
bands 1%. Based on the client's absolute
Management Systems (EMS).
neutrophil count (ANC), the nurse knows that
d) Administer the child's epinephrine. the client’s risk for infection is:

e) Notify the parents. a) No increased risk

4. A nurse encourages a client with an b) Significant risk


immunologic disorder to eat a nutritionally
c) low risk
balanced diet to promote optimal immunologic
function. Which snacks have the greatest d) intermediate risk
probability of stimulating autoimmunity?
8. A client is experiencing an allergic response.
a) Applesauce and dried apricots The nurse should perform the actions in which
order from first to last? All options must be
b) Potato chips and chocolate milk shakes
used.
c) Raisins and carrot sticks
a) Activate the rapid response team.
b) Assess the airway and breathing pattern. C. Early morning stiffness.

c) Notify the health care provider (HCP). D. Rheumatoid nodules.

d) Assess for urticaria. 2. A client with rheumatoid arthritis states, "I


can't do my household chores without
9. A 19-year-old male being tested for multiple
becoming tired. My knees hurt whenever I
allergies develops localized redness and
walk." Which nursing diagnosis would be most
swelling in reaction to a patch skin test. Which
appropriate?
intervention by the nurse would have the
highest priority? A. Activity intolerance related to fatigue and
pain.
a) Notify the primary care provider
B. Self-care deficit related to increasing joint
b) Apply a topical anti-inflammatory cream
pain.
c) Remove the patch and extract from the skin
C. Ineffective coping related to chronic pain.
d) Administer oral diphenhydramine (Benadryl)
D. Disturbed body image related to fatigue and
joint pain.

10. A patient's low hemoglobin and hematocrit 3. Of the clients listed below, who is at risk for
have necessitated a transfusion of packed red developing rheumatoid arthritis (RA)? Select all
blood cells (RBCs). Shortly after the first unit of that apply.
RBCs starts to infuse, the patient develops signs
A. Adults between the ages of 20 and 50 years.
and symptoms of a transfusion reaction. Which
type of hypersensitivity reaction has the patient B. Adults who have had an infectious disease
experienced? with the Epstein-Barr virus.

C. Adults that are of the male gender.

a) Type I D. Adults who possess the genetic link,


specifically HLA-DR4.
b) Type II
E. Adults who also have osteoarthritis.
c) Type III
4. A client is in the acute phase of rheumatoid
d) Type IV
arthritis. Which of the following should the
SAS 33 nurse identify as lowest priority in the plan of
care?

A. Relieving pain.
1. On a visit to the clinic, a client reports the
onset of early symptoms of rheumatoid B. Preserving joint function.
arthritis. The nurse should conduct a
C. Maintaining usual ways of accomplishing
focused assessment for: tasks.

A. Limited motion of joints. D. Preventing joint deformity.

B. Deformed joints of the hands.


5. Which of the following statements should the B. Contact the physician to determine if an
nurse include in the teaching session when alternative examination could be scheduled.
preparing a client for arthrocentesis? Select all
C. Provide a dose of acetaminophen (Tylenol).
that apply.
D. Cancel the examination because of the hard
X-ray table.
A. "A local anesthetic agent may be injected
8. Which of the following should the nurse
into the joint site for your comfort."
assess when completing the history and
B. "A syringe and needle will be used to physical examination of a client diagnosed with
withdraw fluid from your joint." osteoarthritis?

C. "The procedure, although not painful, will A. Anemia.


provide immediate relief."
B. Osteoporosis.
D. "We'll want you to keep your joint active
C. Weight loss.
after the procedure to increase blood flow."
D. Local joint pain.
E. "You will need to wear a compression
bandage for several days after the procedure." 9 During a home health visit you are helping a
patient with gout identify foods in their pantry
they should avoid eating. Select all the foods
6. A client with osteoarthritis will undergo an below the patient should avoid:
arthrocentesis on his painful edematous knee.
A. Sardines
What should be included in the nursing plan of
care? Select all that apply. B. Whole wheat bread
A. Explain the procedure. C. Sweetbreads
B. Administer preoperative medication 1 hour D. Crackers
before surgery.
E. Craft beer
C. Instruct the client to immobilize the knee for
2 days after the surgery. F. Bananas

D. Assess the site for bleeding. 10. Identify which patient below is at MOST risk
for developing gout:
E. Offer pain medication
A. A 56-year-old male who reports consuming
7. A physician orders a lengthy X-ray foods low in purines.
examination for a client with osteoarthritis.
Which of the following actions by the nurse B. A 45-year-old male with a BMI of 40 who
would demonstrate client advocacy? reports taking hydrochlorothiazide and aspirin.

A. Contact the X-ray department and ask the C. A 39-year-old female hospitalized with
technician if the lengthy session can be divided bulimia that has a BMI of 24.
into shorter sessions. D. A 27-year-old female with ulcerative colitis.
MEDSURG – 1 P2 Part 2 (SAS #34-44) c. "I should include more fiber in my diet than a
person who does not have diabetes."

d. "If I use an insulin pump, I will not need to


SAS #34
limit the amount of saturated fat in my diet."
1. The nurse is reviewing laboratory results for
4. Which patient with type 1 diabetes mellitus
the clinic patients to be seen today. Which
would be at the highest risk for developing
patient meets the diagnostic criteria for
hypoglycemic unawareness?
diabetes mellitus?
a. A 58-year-old patient with diabetic
a. A 48-year-old woman with a hemoglobin A1C
retinopathy
of 8.4%
b. A 73-year-old patient who takes propranolol
b. A 58-year-old man with a fasting blood
(Inderal)
glucose of 111 mg/dL
c. A 19-year-old patient who is on the school
c. A 68-year-old woman with a random plasma
track team
glucose of 190 mg/dL
d. A 24-year-old patient with a hemoglobin A1C
d. A 78-year-old man with a 2-hour glucose
of 8.9%
tolerance plasma glucose of 184 mg/dL
5. The nurse is teaching a 60-year-old woman
2. The nurse teaches a 38-year-old man who
with type 2 diabetes mellitus how to prevent
was recently diagnosed with type 1 diabetes
diabetic nephropathy. Which statement made
mellitus about insulin administration. Which
by the patient indicates that teaching has been
statement by the patient requires an
successful?
intervention by the nurse?
a. "Smokeless tobacco products decrease the
a. "I will discard any insulin bottle that is cloudy
risk of kidney damage."
in appearance."
b. "I can help control my blood pressure by
b. "The best injection site for insulin
avoiding foods high in salt."
administration is in my abdomen."
c. "I should have yearly dilated eye
c. "I can wash the site with soap and water
examinations by an ophthalmologist."
before insulin administration."
d. "I will avoid hypoglycemia by keeping my
d. "I may keep my insulin at room temperature
blood sugar above 180 mg/dL."
(75o F) for up to a month."
6. A 54-year-old patient admitted with type 2
3. The nurse instructs a 22-year-old female
diabetes asks the nurse what "type 2" means.
patient with diabetes mellitus about a healthy
What is the most appropriate response by the
eating plan. Which statement made by the
nurse?
patient indicates that teaching was successful?
a. "With type 2 diabetes, the body of the
a. "I plan to lose 25 pounds this year by
pancreas becomes inflamed."
following a high-protein diet."
b. "With type 2 diabetes, insulin secretion is
b. "I may have a hypoglycemic reaction if I drink
decreased, and insulin resistance is increased."
alcohol on an empty stomach."
c. "With type 2 diabetes, the patient is totally c. Set goals for the patient to actively
dependent on an outside source of insulin." participate in managing his diabetes.

d. "With type 2 diabetes, the body produces d. Assume responsibility for all of the patient's
autoantibodies that destroy β-cells in the care to decrease stress level.
pancreas."
10. The nurse has taught a patient admitted
7. The nurse caring for a patient hospitalized with diabetes, cellulitis, and osteomyelitis about
with diabetes mellitus would look for which the principles of foot care. The nurse evaluates
laboratory test result to obtain information on that the patient understands the principles of
the patient's past glucose control? foot care if the patient makes what statement?

a. Prealbumin level a. "I should only walk barefoot in nice dry


weather."
b. Urine ketone level
b. "I should look at the condition of my feet
c. Fasting glucose level
every day."
d. Glycosylated hemoglobin level
c. "I am lucky my shoes fit so nice and tight
8. The nurse has been teaching a patient with because they give me firm support."
diabetes mellitus how to perform self-
d. "When I am allowed up out of bed, I should
monitoring of blood glucose (SMBG). During
check the shower water with my toes."
evaluation of the patient's technique, the nurse
identifies a need for additional teaching when SAS #35
the patient does what?
1. A client diagnosed with pyelonephritis asks
a. Chooses a puncture site in the center of the the nurse "What is the disease?" The nurse's
finger pad. best response "Pyelonephritis is an:

b. Washes hands with soap and water to A. inflammation of the kidney and renal pelvis."
cleanse the site to be used.
B. inflammation of the prostate gland."
c. Warms the finger before puncturing the
C. inflammation of the urethra."
finger to obtain a drop of blood.
D. inflammation of the bladder."
d. Tells the nurse that the result of 110 mg/dL
indicates good control of diabetes. 2. The nurse is planning to teach the client
about the signs and symptoms of a urinary tract
9. The nurse is assigned to the care of a 64-year-
infection. The nurse should include: (Mark all
old patient diagnosed with type 2 diabetes. In
that apply)
formulating a teaching plan that encourages the
patient to actively participate in management A. dysuria.
of the diabetes, what should be the nurse's
initial intervention? B. foul smelling cloudy urine.

a. Assess patient's perception of what it means C. urgency.


to have diabetes. D. back pain.
b. Ask the patient to write down current
knowledge about diabetes.
3. A male client is complaining of urinary C. A 50 y.o. postmenopausal woman
frequency, dysuria, pain, fever and chills for the
D. A 28 y.o. with angina
third time in 9 months. The nurse should expect
which diagnostic test to be ordered since this is 7. Which of the following causes the majority of
the third infection in 9 months? UTI’s in hospitalized patients?
A. Urinalysis A. Lack of fluid intake
B. X-ray of kidneys, ureter and bladder B. Inadequate perineal care
C. Intravenous pyelography C. Invasive procedures
D. Computed tomography of the abdomen D. Immunosuppression
4. Which of the following symptoms do you 8. the most common early sign of kidney
expect to see in a patient diagnosed with acute disease is:
pyelonephritis?
A. Sodium retention
A. Jaundice and flank pain
B. Elevated BUN level
B. Costovertebral angle tenderness and chills
C. Development of metabolic acidosis
C. Burning sensation on urination
D. Inability to dilute or concentrate urine
D. Polyuria and nocturia
9. The nurse is aware that the following
5. A client with pyelonephritis is being laboratory values supports a diagnosis of
discharged from the hospital, and the nurse pyelonephritis?
provides instructions to the client to prevent
recurrence. The nurse determines that the A. Myoglobinuria
client understands the information that was B. Ketonuria
given if the client states an intention to:
C. Pyuria
A. increase fluids for 2 days if signs and
symptoms of a urinary tract infection develop D. Low white blood cell (WBC) count

B. take the prescribed antibiotics until all 10. When examining a female client’s
symptoms subside genitourinary system, Nurse Sandy assesses for
tenderness at the costovertebral angle by
C. return to the physician's office for scheduled placing the left hand over this area and striking
follow-up urine cultures it with the right fist. Normally, this percussion
D. decrease fluid intake if frequent urination technique produces
occurs which sound?
6. Which patient is at greatest risk for A. A flat sound
developing a urinary tract infection (UTI)?
B. A dull sound
A. A 35 y.o. woman with a fractured wrist
C. Hyperresonance
B. A 20 y.o. woman with asthma
D. Tympany
SAS #36 4. A client receiving intravenous chemotherapy
asks the nurse the reason for wearing a mask,
1. The nursing instructor explains the difference
gloves, and gown while
between normal cells and benign tumor cells.
What information does the instructor provide administering drugs to the client. What is the
about these cells? nurse's best response?

a. Benign tumors grow through invasion of a. "These coverings protect you from getting an
other tissue. infection from me."

b. Benign tumors have lost their cellular b. "I am preventing the spread of infection from
regulation from contact inhibition. you to me or any other client here."

c. Growing in the wrong place or time is typical c. "The policy is for any nurse giving these drugs
of benign tumors. to wear a gown, gloves, and mask."

d. The loss of characteristics of the parent cells d. "The clothing protects me from accidentally
is called anaplasia. absorbing these drugs."

2. A nurse has taught a client about dietary 5. The nurse is administering a combination of
changes that can reduce the chances of three different antineoplastic drugs to a patient
developing cancer. What statement by the who has metastatic breast cancer. Which
client indicates the nurse needs to provide statement best describes the rationale for
additional teaching? combination therapy?

a. "Food high in vitamin A and vitamin C are a. There will be less nausea and vomiting.
important."
b. Increased cancer-cell killing will occur.
b. "I'll have to cut down on the amount of
c. The drugs will prevent metastasis.
bacon I eat."
d. Combination therapy reduces the need for
c. "I'm so glad I don't have to give up my juicy
radiation therapy.
steaks."
6. One patient has cancer of the bone; another
d. "Vegetables, fruit, and high-fiber grains are
has cancer in the connective tissues of the thigh
important."
muscles; a third patient has cancer in the
3. A nurse is participating in primary prevention vascular tissues. These patients have a type of
efforts directed against cancer. In which activities is tumor referred to as a
this nurse most likely to engage? (Select all that
apply.) a. sarcoma.

a. Demonstrating breast self-examination methods b. leukemia


to women
c. carcinoma.
b. Instructing people on the use of chemoprevention
d. lymphoma.
c. Providing vaccinations against certain cancers

d. Screening teenage girls for cervical cancer

e. Teaching teens the dangers of tanning booths


7. A patient who has cancer is about to begin 10. Which manifestation of an oncologic
chemotherapy. The patient asks the nurse why emergency requires the nurse to contact the
two chemotherapeutic agents are being used health care provider immediately?
instead of just one. Which response by the
a. New onset of fatigue
nurse is correct?
b. Edema of arms and hands
a. "The drugs may be given in less toxic doses if
two drugs are used." c. Dry cough
b. "Two agents used together can have d. Weight gain
synergistic effects."
SAS #37
c. "Use of two drugs will increase tumoricidal
activity in the G0 phase of the cell."

d. "Using two agents will shorten the length of 1. The nurse is taking the social history of a
time chemotherapy is needed." client diagnosed with Small Cell Lung Cancer.
Which information is significant for this
8. The nurse is teaching a patient who will begin disease?
receiving targeted therapy for cancer. The
patient asks how targeted therapy differs from a. Worked with asbestos for a short time many
other types of chemotherapies. The nurse will years ago.
explain that targeted therapy b. Has no family History of this type of lung
a. damages cancer cell DNA to prevent cell cancer.
replication. c. Has numerous tattoos on upper and lower
b. directly kills or damages cancerous cells. arms.

c. interferes with specific molecules in cancer d. Has smoked 2 packs of cigarettes/day for 20
cells. years.

d. prevents metastasis of cancer cells. 2. The nurse writes a problem of 'impaired gas
exchange' for a client diagnosed with cancer of
9. The registered nurse is teaching a nursing the lung. Which interventions should be
student about the importance of observing for included for the plan of care? Select all that
bone marrow suppression during apply.
chemotherapy. Select the person who displays
bone marrow suppression. a. Apply O2 via nasal cannula.

a. Client with hemoglobin of 7.4 and hematocrit b. Have the dietician plan for 6 small meals per
of 21.8 day.

b. Client with diarrhea and potassium level of c. Place the client in respiratory isolation.
2.9 mEq/L d. Assess vital signs for fever.
c. Client with 250,000 platelets e. Listen to lung sounds every shift.
d. Client with 5000 white blood cells/mm3
3. The nurse is discussing cancer statistics with a the client indicates that more teaching is
group from the community. Which information needed?
about death rates from lung cancer is accurate?
a. It doesn't matter if I smoke now. I already
a. Lung Cancer is the number 2 cause of cancer have cancer.
deaths in both men and women.
b. I should see the oncologist at my scheduled
b. Lung Cancer is the number 1 cause of cancer appointment
deaths in both men and women.
c. If I begin to run a fever, I should notify my
c. Lung Cancer deaths are not significant in Health Care Provider.
relation to other cancers.
d. I should plan for periods of rest throughout
d. Lung Cancer deaths have continued to the day.
increase in the male population
7. The client is admitted to the outpatient
4. The client diagnosed with lung cancer has surgery center for a bronchoscopy to rule out
been told that the cancer has metastasized to lung cancer. Which info should the nurse teach?
the brain. Which intervention should the nurse
a. The test will confirm the MRI results.
implement?
b. The client can eat and drink immediately
a. Discuss implementing an advance directive.
after the test.
b. Explain the use of chemotherapy for brain
c. The Health Care Provider can do a biopsy of
involvement.
the tumor through the scope.
c. Teach the client to discontinue driving.
d. There is no discomfort associated with this
d. Have the significant other make decisions for procedure.
the client.
8. Which clinical manifestation would the nurse
5. The client diagnosed with lung cancer is in an expect to find in newly diagnosed intrinsic Lung
investigational program and receiving a vaccine Cancer?
to treat the cancer. Which information
a. Dysphagia
regarding investigational regimens should the
nurse teach? b. Foul smelling breath
a. Investigational regimens provide a better c. Hoarseness
chance of survival for the client.
d. Weight loss
b. Investigational treatments have not been
proved helpful to clients. 9. A patient who smokes tells the nurse, "I want
to have a yearly chest x-ray so that if I get
c. Clients will be paid to participate in a cancer, it will be detected early." Which
investigational program. response by the nurse is most appropriate?
d. Only clients that are dying qualify for a. "Chest x-rays do not detect cancer until
investigational treatments. tumors are already at least a half-inch in size."
6. The client diagnosed with Lung Cancer is b. "Annual x-rays will increase your risk for
being discharged. Which statement made by cancer because of exposure to radiation."
c. "Insurance companies do not authorize yearly B. Air bubbles should be expelled on wet
x-rays just to detect early lung cancer." cotton.

d. "Frequent x-rays damage the lungs and make C. Label the hanging IV bottle with
them more susceptible to cancer." “ANTINEOPLASTIC CHEMOTHERAPY” sign.

10. The nurse has identified the nursing D. Vent vials after mixing.
diagnosis of imbalanced nutrition: less than
3. Neoplasm can be classified as either benign
body requirements related to altered taste
or malignant. The following are characteristics
sensation in a patient with lung cancer who has
of malignant tumor apart
had a 10% loss in weight. An appropriate
nursing intervention that addresses the etiology from:
of this problem is to
A. Metastasis
a. provides foods that are highly spiced to
stimulate the taste buds. B. Infiltrates surrounding tissues

b. avoids presenting foods for which the patient C. Encapsulated


has a strong dislike. D. Poorly differentiated cells
c. adds strained baby meats to foods such as 4. On a clinic visit a client who has a relative
soups and casseroles. with cancer, is asking about the warning signs
d. teaches the patient to eat whatever is that may relate to cancer. The nurse correctly
nutritious since food is tasteless. identifies the warning signs of cancer by
responding:
SAS #38
A. “If a sore healing took a month or more to
1. A patient with liver cancer has a nursing heal, cancer should be suspected.”
diagnosis of imbalanced nutrition: less than
body requirements r/t anorexia and inadequate B. “Presence of dry cough is one of the warning
food intake. An appropriate midday snack for signs of cancer.”
the patient would be C. “A lump located only in the breast area may
A. peanut butter and salt free crackers suggest the presence of cancer.”

B. a fresh tomato sandwich with salt free butter D. “Sudden weight loss of unexplained etiology
can be a warning sign of cancer.”
C. popcorn with salt free butter and herbal
seasoning 5. Skin reactions are common in radiation
therapy. Nursing responsibilities on promoting
D. canned chicken noodle soup with low protein skin integrity should be promoted apart from:
bread
A. Avoiding the use of ointments, powders and
2. The nurse is preparing Cyclophosphamide lotion to the area
(Cytoxan). Safe handling of the drug should be
implemented to protect thebnurse from injury. B. Using soft cotton fabrics for clothing
Which of the following action by the nurse C. Washing the area with a mild soap and water
should be corrected? and patting it dry not rubbing it.
A. The nurse should wear mask and gloves.
D. Avoiding direct sunshine or cold. B. 3 minutes

6. Nausea and vomiting is an expected side C. 5 minutes


effect of chemotherapeutic drug use. Which of
D. 10 minutes
the following drug should be administered to a
client on chemotherapy to prevent nausea and 10. A client is taking Cyclophosphamide
vomiting? (Cytoxan) for the treatment of lymphoma. The
nurse is very cautious in administering the
A. Metochlopramide (Metozol)
medication because this drug poses the fatal
B. Succimer (Chemet) side effect of:

C. Anastrazole (Arimidex) A. Alopecia

D. Busulfan (Myleran) B. Myeloma

7. Radiation protection is very important to C. CNS toxicity


implement when performing nursing
D. Hemorrhagic cystitis
procedures. When the nurse is not performing
any nursing procedures what distance should SAS #39
be maintained from the client?
1. A patient diagnosed with thyroid cancer has
A. 1 feet undergone a thyroidectomy after
thyroidectomy, which of the following is the
B. 2 feet
priority assessment to observe laryngeal nerve
C. 2.5 feet damage?

D. 3 feet A hoarseness of voice

8. The following are teaching guidelines B difficulty in swallowing


regarding radiation therapy except:
C tetany
A. The therapy is painless
D fever
B. To promote safety, the client is assisted by
2. For the first 72 hours after thyroidectomy
therapy personnel while the machine is in
surgery, nurse Jamie would assess the female
operation.
client for Chvostek's sign and Trousseau's sign
C. The client may communicate all his concerns because they indicate which of the following?
or needs or discomforts while the machine is
A. Hypocalcemia
operating.
B. Hypercalcemia
D. Safety precautions are necessary only during
the time of actual irradiation. C. Hypokalemia
9. Contact of client on radiation therapy should D. Hyperkalemia
be limited only to how many minutes to
promote safety of the therapy personnel? 3. Early this morning, a female client had a
subtotal thyroidectomy. During evening rounds,
A. 1 minute nurse Tina assesses the client, who now has
nausea, a temperature of 105° F (40.5° C),
tachycardia, and extreme restlessness. What is C. Tumors form when carcinogens invade the
the most likely cause of these signs? gene structure of the cell in the latency phase.

A. Diabetic ketoacidosis D. Nutrition of cancer cells is provided by tumor


angiogenesis factor (TAF) in the promotion
B. Thyroid crisis
stage.
C. Hypoglycemia
7 Which potential side effects should be
D. Tetany included in the teaching plan for a client
undergoing radiation therapy for thyroid
4. In the administration of a drug such as cancer? select all that apply.
levothyroxine (Synthroid), the nurse should
teach the client: A. Fatigue

A. That therapy typically lasts about 6 months. B. Changes in color of hair

B. That weekly laboratory tests for T4 levels will C. Change in taste


be required.
D. Changes in skin of the neck
C. To report weight loss, anxiety, insomnia, and
E. Difficulty swallowing
palpitations.
8. The nurse has received in report that the
D. That the drug may be taken every other day
client receiving chemotherapy for thyroid
if diarrhea occurs.
cancer has severe neutropenia. Which of the
5. Which precaution is most important for the following does the nurse plan to implement?
nurse to teach a client receiving radiation Select all that apply.
therapy for head and neck cancer?
A. Assess for fever.
A. Avoid eating red meat during treatment.
B. Observe for bleeding.
B. Pace your leisure activities to prevent fatigue.
C. Administer pegfilgrastim (Neulasta).
C. See your dentist twice yearly for the rest of
D. Do not permit fresh flowers or plants in the
your life.
room.
D. Avoid using headphones or headsets until
E. Do not allow his 16-year-old son to visit.
your hair grows back.
F. Teach the client to omit raw fruits and
6. The registered nurse is teaching a group of
vegetables from his diet.
nursing students about malignant
transformation. Which statement about the 9. Which medication does the nurse plan to
process of malignant transformation is true? administer to a client before chemotherapy for
thyroid cancer to decrease the incidence of
A. Mutation of genes is an irreversible event
nausea?
that always leads to cancer development in the
initiation phase. A. Morphine

B. Insulin and estrogen enhance the division of B. Ondansetron (Zofran)


an initiated cell during the promotion phase.
C. Naloxone (Narcan)
D. Diazepam (Valium) c. are simply an overgrowth of normal cells.

10. The nurse is caring for a patient with thyroid d. frequently recur in the same site
cancer suffering from anorexia secondary to
3. The community nurse is conducting a health
chemotherapy. Which of the following
promotion program and the topic of the
strategies would be most appropriate for the
discussion relates to the risk factors for gastric
nurse to use to increase the patient's nutritional
cancer. Which risk factor, if identified by a
intake?
client, indicates a need for further discussion?
A. Increase intake of liquids at mealtime to
a. smoking
stimulate the appetite.
b. a high-fat diet
B. Serve three large meals per day plus snacks
between each meal. c. foods containing nitrates
C. Avoid the use of liquid protein supplements d. a diet of smoked, highly salted, and spiced
to encourage eating at mealtime. food
D. Add items such as skim milk powder, cheese, 4. A gastrectomy is performed on a client with
honey, or peanut butter to selected foods. gastric cancer. In the immediate postoperative
period, the nurse notes bloody drainage from
the nasogastric tube. Which of the following is
SAS #40 the appropriate nursing intervention?

1. A gastrectomy is performed on a client with a. notify the physician


gastric cancer. In the immediate postoperative
b. measure abdominal girth
period, the
c. irrigate the nasogastric tube
nurse notes bloody drainage from the
nasogastric tube. Which of the following is the d. continue to monitor the drainage
appropriate nursing intervention?
5. A female client with cancer is scheduled for
a. notify the physician radiation therapy. The nurse knows that
radiation at any treatment site may cause a
b. measure abdominal girth
certain adverse effect. Therefore, the nurse
c. irrigate the nasogastric tube should prepare the client to expect:

d. continue to monitor the drainage a. hair loss.

2. While being prepared for a biopsy; the b. stomatitis.


patient asks the nurse what the difference is
c. fatigue.
between a benign tumor and a malignant
tumor. The nurse explains that a benign tumor d. vomiting.
differs from a malignant tumor in those benign
tumors: 6. A gastrectomy is performed on a client with
gastric cancer. In the immediate postoperative
a. do not cause damage to adjacent tissue. period, the nurse notes
b. do not spread to other tissues and organs.
bloody drainage from the nasogastric tube. c.) Learn to self-administer enteral feedings
Which of the following is the appropriate every 4 hours.
nursing intervention.
d.) Maintain adequate nutrition through oral or
a. Notify the physician parenteral feedings

b. Measure the abdominal girth 10. Teaching a client who has had recent bowel
surgery how to facilitate the expulsion of feces
c. Irrigate the nasogastric tube
may include the process of increasing intra-
d. Continue to monitor the drainage abdominal pressure. Which of the following
best matches this process?
7. An optimal teaching plan for an outpatient
with stomach cancer receiving radiation therapy a. Crede's maneuver.
should include information about
b. Valsalva's maneuver.
a. cancer support groups.
c. Heimlich's maneuver.
b. avitaminosis, ostomy care, and community
d. Epley's maneuver
resources.
SAS #41
c. prosthetic devices, skin conductance, and
grief counseling. 1. A nurse assesses clients at a community
health center. Which client is at highest risk for
d. wound and skin care, nutrition, drugs, and
the development of colorectal cancer?
community resources
a. A 37-year-old who drinks eight cups of coffee
8. Chemotherapy is one of the therapeutic
daily
modalities for cancer. This treatment is
contraindicated to which of the following b. A 44-year-old with irritable bowel syndrome
conditions? (IBS)

a. Recent surgery c. A 60-year-old lawyer who works 65 hours per


week
b. Pregnancy
d. A 72-year-old who eats fast food frequently
c. Bone marrow depression
2. A nurse assessing a client with colorectal
d. All of the above
cancer auscultates high-pitched bowel sounds
9. Which one of the following expected and notes the presence of visible peristaltic
outcomes about nutrition would be appropriate waves. Which action should the nurse take?
for a client who has had a total gastrectomy for
a. Ask if the client is experiencing pain in the
gastric cancer? The client will:
right shoulder.
a.) Regain any weight lost within 4 weeks of the
b. Perform a rectal examination and assess for
surgical procedure.
polyps.
b.) Eat three full meals a day without
c. Contact the provider and recommend
experiencing gastric complications.
computed tomography.
d. Administer a laxative to increase bowel b. The enterostomal therapist will be able to
movement activity. answer all of your questions.

3. A nurse prepares a client for a colonoscopy c. I will make a referral to the United Ostomy
scheduled for tomorrow. The client states, "My Associations of America.
doctor told me that the fecal occult blood test
d. You'll find that most people with colostomies
was negative for colon cancer. I don't think I
don't want to talk about them.
need the colonoscopy and would like to cancel
it." How should the nurse respond? 6. A nurse teaches a client who is recovering
from a colon resection. Which statement should
a. Your doctor should not have given you that
the nurse include in these
information prior to the colonoscopy.
clients plan of care?
b. The colonoscopy is required due to the high
percentage of false negatives with the blood a. You may experience nausea and vomiting for
test. the first few weeks.
c. A negative fecal occult blood test does not b. Carbonated beverages can help decrease acid
rule out the possibility of colon cancer. reflux from anastomosis sites.
d. I will contact your doctor so that you can c. Take a stool softener to promote softer stools
discuss your concerns about the procedure. for ease of defecation.
4. A nurse cares for a client newly diagnosed d. You may return to your normal workout
with colon cancer who has become withdrawn schedule, including weight lifting.
from family members. Which action should the
nurse take? 7. A nurse teaches a client who is at risk for
colon cancer. Which dietary recommendation
a. Contact the provider and recommend a should the nurse teach this client?
psychiatric consult for the client.
a. Eat low-fiber and low-residual foods.
b. Encourage the client to verbalize feelings
about the diagnosis. b. White rice and bread are easier to digest.

c. Provide education about new treatment c. Add vegetables such as broccoli and
options with successful outcomes. cauliflower to your new diet.

d. Ask family and friends to visit the client and d. Food high in animal fat help to protect the
provide emotional support. intestinal mucosa.

5. A nurse cares for a client with colon cancer 8. A nurse cares for a client who has a family
who has a new colostomy. The client states, "I history of colon cancer. The client states, "My
think it would be helpful to talk with someone father & brother had colon cancer. What is the
who has had a similar experience." How should chance that I will get cancer?" How should the
the nurse respond? nurse respond?

a. I have a good friend with a colostomy who a. If you eat a low-fat and low-fiber diet, your
would be willing to talk with you. chances decrease significantly.
b. You are safe. This is an autosomal dominant SAS #42
disorder that skips generations.
1. The client frequently finds lumps in her
c. Preemptive surgery and chemotherapy will breasts, especially around the time of her
remove cancer cells and prevent cancer. period. Which information should the nurse
teach the client regarding breast self-care?
d. You should have a colonoscopy more
frequently to identify abnormal polyps early. A. This is a benign process that does not need
follow up.
9. After teaching a client who is recovering from
a colon resection, the nurse assesses the client's B. Eliminate chocolate and caffeine from diet.
understanding. Which statements by the client
C. Practice breast self-exam monthly.
indicate a correct understanding of the
teaching? (SATA) D. This is how breast cancer starts and she
needs surgery.
a. I must change the ostomy appliance daily and
as needed. 2. The client diagnosed with breast cancer is
considering whether to have a lumpectomy or a
b. I will use warm water and a soft washcloth to
more invasive procedure, a modified radical
clean around the stoma.
mastectomy. Which of the following should the
c. I might start bicycling and swimming again nurse discuss?
once my incision has healed.
A. Ask if she is afraid of general anesthesia.
d. Cutting the flange will help it fit snugly
B. Ask how she feels about radiation and
around the stoma to avoid skin breakdown.
chemotherapy.
e. I will check the stoma regularly to make sure
C. Tell her that she will need reconstruction
that it stays a deep red color.
with either procedure.
f. I must avoid dairy products to reduce gas and
D. Find out if she has Breast Cancer in her
odor in the pouch.
family.
10. A client has late-stage colon cancer with
3. The client has undergone a wedge resection
metastasis to the spine and bones. Which
for cancer on the left breast. Which discharge
nursing intervention does the nurse add to the
instruction should the nurse teach?
care plan to address a priority problem?
A. Don't life more than 5 lbs. with left hand until
a. Provide six small meals and snacks daily.
released by Health care provider
b. Offer the client prune juice twice a day.
B. The cancer has been totally removed and no
c. Ensure that the client gets adequate rest. follow up therapy is required.

d. Give the client pain medications around the C. Client should empty Hemovac every 12
clock. hours.

D. Client should arrange for an appointment


with a plastic surgeon for reconstruction.
4. The nurse is teaching a class on breast health include? Select all that apply:
to a group of ladies at the senior center. Which
A. Notify health care provider of temperature of
is the most important risk factor to emphasize
100 F
to this group?
B. Carry large purses and bundles with the right
A. Find out about family History of Breast
hand.
Cancer
C. Do not go to church or anywhere with
B. Men at this age can get breast cancer and
crowds.
should also be screened.
D. Try to keep the arm as still as possible until
C. Monthly self-exam is the key to early
seen by health care provider.
detection.
E. Have a mammogram of the right and left
D. The older a woman gets, the greater the
breast yearly
chance of developing Breast Cancer.
8. The client who has had a mastectomy tells
5. The client who is scheduled for breast biopsy
the nurse, "My husband will leave me now that
with sentinel node dissection sates, "I don't
I am not a whole woman anymore." Which
understand. What does a sentinel node biopsy
response is therapeutic?
do?"
A. Are you afraid that your husband will not find
A. A dye is injected into the tumor and traced to
you sexually appealing?
determine spread of cells.
B. Your husband should be grateful that you will
B. The surgeon removes the nodes that drain
be able to live and be with him.
the diseased portion of the breast.
C. Maybe your husband would like to attend a
C. The nodes that can be felt manually will be
support group for spouses.
removed and sent to pathology.
D. You don't know that's true. Give him a
D. A visual inspection of the lymph nodes will e
chance.
made while the client is sleeping.
9. The client has been diagnosed with BC. What
6. The client is 4 months pregnant and finds a
is the most appropriate referral for the nurse to
lump in her breast and the biopsy is positive for
make?
stage II Breast Cancer. Which Treatment should
the nurse anticipate? A. Hospital social worker
A. A lumpectomy to be performed after the B. CanSurmount
baby is born.
C. Reach to Recovery
B. A modified radical mastectomy.
D. ICanCope
C. Radiation therapy to chest wall only.
10 The client has undergone a wedge resection
D. Chemo only until baby is born. for cancer on the left breast. Which discharge
instruction should the nurse teach?
7. The client is being discharged after a left
modified radical mastectomy. Which discharge A. Don't life more than 5 lbs. with left hand until
instructions should the nurse released by health care provider
B. The cancer has been totally removed and no D Dose-limiting side effects
follow-up therapy is required.
4. The nurse corrects the nursing student when
C. Client should empty Hemovac every 12 caring for a client with neutropenia secondary
hours. to chemotherapy in which circumstance?

D. Client should arrange for an appt. with a A The student scrubs the hub of IV tubing
plastic surgeon for reconstruction. before administering an antibiotic.

SAS #43 B The nurse overhears the student explaining to


the client the importance of handwashing.
1. The nurse is caring for a client with end-stage
ovarian cancer who needs clarification on the C The student teaches the client that symptoms
purpose of palliative surgery. Which outcome of neutropenia include fatigue and weakness.
should the nurse teach the client is the goal of
D The nurse observes the student providing oral
palliative surgery?
hygiene and perineal care.
A Cure of the cancer
5. Which client problem does the nurse set as
B Relief of symptoms or improved quality of life the priority for the client experiencing
chemotherapy-induced peripheral neuropathy?
C Allowing other therapies to be more effective
A Potential for lack of understanding related to
D Prolonging the client's survival time
side effects of chemotherapy
2. Which statement made by a client allows the
B Potential for injury related to sensory and
nurse to recognize whether the client receiving
motor deficits
brachytherapy for ovarian cancer understands
the treatment plan? C Potential for ineffective coping strategies
related to loss of motor control
A "I may lose my hair during this treatment."
D Altered sexual function related to erectile
B "I must be positioned in the same way during
dysfunction
each treatment."
6. Which finding alarms the nurse when caring
C "I will have a radioactive device in my body
for a client receiving chemotherapy who has a
for a short time."
platelet count of 17,000/mm3?
D "I will be placed in a semiprivate room for
A Increasing shortness of breath
company."
B Diminished bilateral breath sounds
3.When caring for a client receiving
chemotherapy, the nurse plans care during the C Change in mental status
nadir of bone marrow activity to prevent which
D Weight gain of 4 pounds in 1 day
complication?
7. Which medication does the nurse plan to
A Drug toxicity
administer to a client before chemotherapy to
B Polycythemia decrease the incidence of nausea and vomiting?

C Infection A Morphine
B Ondansetron (Zofran) D Breast tenderness

C Naloxone (Narcan) E Weight loss

D Diazepam (Valium) F Deep vein thrombosis

8. When caring for the client receiving cancer


chemotherapy, which signs or symptoms
SAS #44
related to thrombocytopenia should the nurse
report to the health care provider? (Select all 1. A 33-yr-old patient has recently been
that apply.) diagnosed with stage II cervical cancer. Which
statement by the nurse best explains the
A Bruises
diagnosis?
B Fever
a. "The cancer is found at the point of origin
C Petechiae only."

D Epistaxis b. "Tumor cells have been identified in the


cervical region."
E Pallor
c. "The cancer has been identified in the cervix
9. The nurse has received in report that a client
and the liver."
receiving chemotherapy has severe
neutropenia. Which interventions does the d. "Your cancer was identified in the cervix and
nurse plan to implement? (Select all that apply.) has limited local spread."

A Assess for fever. 2. The patient is told that her adenoma tumor is
not encapsulated but has normally
B Observe for bleeding.
differentiated cells and surgery will be needed.
C Administer pegfilgrastim (Neulasta). The patient asks the nurse what this means.
What should the nurse tell the patient?
D Do not permit fresh flowers or plants in the
room. a. It will recur.

E Do not allow the client's 16-year-old son to b. It has metastasized.


visit.
c. It is probably benign.
F Teach the client to omit raw fruits and
d. It is probably malignant.
vegetables from the diet.
3. Which statement by the nurse most
10. The nurse is teaching a client who is
facilitates patient cancer prevention during the
receiving an antiestrogen drug about the side
promotion stage of cancer development?
effects she may encounter. Which side effects
does the nurse include in the discussion? (Select a. "Exercise every day for 30 minutes."
all that apply.)
b. "Follow smoking cessation
A Heavy menses recommendations."

B Smooth facial skin c. "Following a vitamin regime is highly


recommended."
C Hyperkalemia
d. "I recommend excision of the cancer as soon f. Teach them about the different types of
as possible." cancer that could be diagnosed

4. The laboratory reports that the cells from the 7. The patient is receiving an IV vesicant
patient's tumor biopsy are grade II. What chemotherapy drug. The nurse notices swelling
should the nurse know about this histologic and redness at the site. What should the nurse
grading? do first?

a. Cells are abnormal and moderately a. Ask the patient if the site hurts.
differentiated.
b. Turn off the chemotherapy infusion.
b. Cells are very abnormal and poorly
c. Call the ordering health care provider.
differentiated.
d. Administer sterile saline to the reddened
c. Cells are immature, primitive, and
area.
undifferentiated.
8. When caring for the patient with cancer,
d. Cells differ slightly from normal cells and are
what does the nurse understand as the
well-differentiated.
response of the immune system to antigens of
5. Patients may reduce the risk of developing the malignant cells?
cancer using health promotion strategies.
a. Metastasis
Identify strategies which can reduce the risk of
developing cancer (select all that apply.). b. Tumor angiogenesis
a. Control weight c. Immunologic escape
b. Genetic testing d. Immunologic surveillance
c. Immunizations 9. The patient is being treated with
brachytherapy for cervical cancer. What factors
d. Use sunscreen
must the nurse be aware of to protect herself
e. Stop smoking when caring for this patient?

f. Limit alcohol intake a. The medications the patient is taking

6. The patient and his family are upset that the b. The nutritional supplements that will help the
patient is going through procedures to diagnose patient
cancer. What nursing actions should the nurse
c. How much time is needed to provide the
use first to facilitate their coping with this
patient's care
situation (select all that apply.)?
d. The time the nurse spends at what distance
a. Maintain hope
from the patient
b. Exhibit a caring attitude
10. A client, age 41, visits the gynecologist. After
c. Plan realistic long-term goals examining her, the physician suspects cervical
cancer. The nurse
d. Give them anti-anxiety medications

e. Be available to listen to fears and concerns


reviews the client’s history for risk factors for
this disease. Which history finding is a risk
factor for cervical cancer?

a. Onset of sporadic sexual activity at age 17

b. Spontaneous abortion at age 19

c. Pregnancy complicated with eclampsia at age


27

d. Human papillomavirus infection at age 32

You might also like